Clinical Quiz Series For Medical Students And Board Candidates[Paper 8] PDF

Clinical Quiz Series For Medical Students And Board Candidates[Paper 8] PDF

Clinical Quiz Series For Medical Students And Board Candidates[Paper 8] PDF

Please support this website by 1 , 3 or 5 $$

   Support NOW

Files Size:: 1.3 MB

Free Download link

DR. FAHMI KHAN: www.fakhanqal.com

Unlimited permission is granted free of charge to print or photocopy all pages of this work for educational, not-for-profit use by faculty members, students or board candidates.

Clinical Quiz series for medical students and board candidates

Paper 8: Contains 120 best of five questions; 60 MCQs; 2 0 slides and answer key

Dr.Fahmi Khan

—————————————————–Page 1—————————————————–

A: For each question below choose the single best answer

1. A 65-year-old obese man presents with recurrent episodes of cough. His wife said his coughing attacks disturb her sleep almost every night. He noticed that his voice is hoarse in the morning. He has never smoked and has no history of cardiac or respiratory problems. There is no history of allergy and the chest X-ray is normal. The procedure most likely to yield crucial diagnostic information is A) Pulmonary function tests

B) 24-hour pH monitoring of the lower oesophagus C) High-resolution CT scan of the chest D) Barium swallow

E) Indirect laryngoscopy

2. A 38-year-old man has a six-month history of diarrhea, abdominal pain and tenesmus. Flexible sigmoidoscopy reveals proctitis. A biopsy specimen shows acute and chronic inflammation. Each of the following statements is true about this disorder, EXCEPT

A) There is an increased risk of gastrointestinal malignancy B) Other family members are likely to develop the same illness

C) Steatorrhea is likely to develop as a result of pancreatic insufficiency D) The likelihood of renal stones will increase E) The likelihood of gallstones will increase

3 . A 56-year-old man is referred with dyspnea and cough productive of putrid green sputum of four weeks duration. The chest X-ray shows cavity formation in the left lower lobe. He has advanced motor neurone disease. This clinical picture is most likely to be caused by infection with

A) Legionella pneumophila B) Mycoplasma pneumoniae C) Streptococcus pneumoniae D) Anaerobic bacteria E) Common cold virus

4. A 50-year-old publican is referred with anuria. The initial renal function test showed the urea at 45 mmol/I and the creatinine at 480 µ moI/l. A urine sample from a catheter was strongly positive for myoglobin. This clinical picture can develop as a result of each of the following clinical situations EXCEPT

A) Alcohol ingestion B) Hypokalaemia

C) Barbiturate overdose D) Hypophosphataemia E) Hypocalcaemia

5 . Which of the follinwg is NOT a criterion for obesity hypoventilion syndrome (OHS)?

A) Obesity (BMI > 30 Kg/m2)

B) Neck circumference of > 48 cm C) Daytime hypoventilation

D) Hypercapnia with PaCO2 > 45 mm Hg E) Hypoxia with PaO2 < 70 mm Hg

2

—————————————————–Page 2—————————————————–

6. A 28-year-old woman presented to the emergency department 12 days after childbirth with worsening headache and progressive visual loss. She had spontaneous pregnancy and uneventful term delivery. Three weeks before delivery headache developed which was persistent and moderately severe. Her sister had chronic autoimmune thyroiditis. On physical examination weight 72.5kg height 165 cm, BP 100/70 mmHg. Neurological examination was notable for bitemporal hemianopia. Laboratory data includes: Free T4 0.5 ng/dl (N 0.9-0.2): TSH 0.2 mu/I (0.4-4.0): Morning serum cortisol 3ug/dl (N 5-25): Morning plasma ACTH 6pg/ml (N 12- 48): Prolactin 45.3ng/ml (N 5-20): MRI scan of the pituitary showed 1 7×1.4×1.9cm

homogenously enhancing sellar mass with suprasellar extension. The MOST LIKELY diagnosis is:

A) Prolactin secreting pituitary adenoma.

B) Clinically nonfunctioning pituitary adenoma. C) Autoimmune hypophysitis D) Pituitary tumor apoplexy.

7 . A 66-year-old woman with poorly controlled type 2 diabetes presents with a 2-day history of severe pain in the right flank, nausea and fever that come and go. On examination, the patient appears unwell, sweaty and has visible rigors with a temperature of 38°C. The patient denies any recent travel. Urine dipstick is positive for protein, blood, leukocytes and nitrates. A CT scan of the abdomen reveals gas in the renal parenchyma area. The MOST LIKELY diagnosis is:

A) Renal stones

B) Renal infarction

C) Diabetic nephropathy D) Renal TB

E) Pyelonephritis

8. A 75-year-old woman is admitted with left lower lobe pneumonia. She was given a course of cephalosporin antibiotics and she improved. However, at the end of the course she started to have diarrhea, passing a watery stool. Investigations confirmed the diagnosis of

pseudomembranous colitis. Each of the following oral medications is beneficial in treating this condition, EXCEPT

A) Vancomycin B) Bacitracin

C) Colestyramine D) Metronidazole E) Gentamicin

9. A 15-year-old girl presents with diplopia after prolonged reading and ptosis that worsens in the afternoon. On examination, she is noted to have bilateral ptosis, impaired extraocular muscle movements, facial weakness, and generalized hypotonia and weakness increasing with repetition. Which of the following is the best diagnostic test for this disorder?

A) CT of the brain

B) Electromyography C) Lumbar puncture D) Muscle biopsy

E) Nerve conduction velocity

3

—————————————————–Page 3—————————————————–

10. In pituitary adenoma all the following statements are correct EXCEPT

A) Prolactin secreting adenoma is the most common.

B) In acromegaly macroadenoma is commonly found at presentation. C) Cushing’s disease is more common in females.

D) Central diabetes insipidus is often found at presentation.

E) Dopamine agonists are highly effective in tumor shrinking in patients with prolactinoma.

11. A 52-year-old woman has diarrhea, wheezing, and the flushing reactions shown here, which seemed to be associated with emotion and alcohol ingestion. Which one of the following will be MOST EFFECTIVE for alleviating the symptoms?

A) β -Adrenergic blockade B) Psychotherapy

C) Bulk fiber supplementation D) Somatostatin analogues E) Systemic corticosteroids

12. Whilst looking through the past medical history notes of one of your inpatients (admitted for ENT surgery) you note that she suffers with fibrosing alveolitis. Causes of this include all of the following EXCEPT

A) Cryptogenic

B) Rheumatoid arthritis C) Sjögren’s syndrome D) Ulcerative colitis E) Aspergillus

13 . A 20-year-old man comes to your office with a dog bite to his left thigh received after he and a friend taunted a neighbor’s dog. He reports that the bite occurred about 36 hours ago and only came to your office after coworkers informed him that dog bites frequently become infected. His temperature is 37.0 C (98.6 F), blood pressure is 110/70 mm Hg, pulse is 63/min, and

respirations are 13/min. On physical examination you notice a shallow abrasion on his left thigh, which is mildly tender. There is no surrounding edema or erythema. The most appropriate management of this patient is to A) Administer a rabies vaccination B) Prescribe amoxicillin-clavulanate C) Prescribe clindamycin D) Prescribe penicillin

E) Provide local wound care without antibiotic therapy

14. A 19-year-old student presents with acute shortness of breath and haemoptysis. She reported left loin pain and was found to have haematuria 2 weeks ago. She is under regular review by the renal physicians for nephrotic syndrome. Ultrasound of the kidney shows left renal vein thrombosis and a computed tomography pulmonary angiogram confirms acute pulmonary embolism. Which one of the following is the MOST LIKELY cause of this patient’s increased tendency for developing venous thrombosis?

A) Factor V Leiden mutation

B) High levels of anticardiolipin antibodies C) Protein S deficiency D) Protein C deficiency

E) Antithrombin III deficiency

4

—————————————————–Page 4—————————————————–

15. Which one of the following conditions is MOST LIKELY to be associated with pyoderma gangrenosum? A) Tuberculosis B) Leprosy

C) Chronic myeloid leukemia D) Sulphonamide therapy E) Cushing’s syndrome

16. A 59-year-old female with poorly controlled hypertension on three antihypertensive medications including a beta-blocker, angiotensin converting enzyme inhibitor, and a calcium channel blocker, presents for a routine surgery. At her preoperative visit, she is instructed to hold her home ACE inhibitor the day of surgery. She then undergoes an otherwise uncomplicated cholecystectomy with intraoperative cholangiogram. However, in the postoperative care unit, she develops hypertension with readings o 200/140. Her other vital signs are within the normal range. The anesthesia provider pefrorms an eye exam and notes papilledema. Nitroprusside is administered IV or treatment. Regarding the immediate management of this patient’s malignant hypertension, which of the following is correct? A) Goal BP is reduction to normal rage o SBP < 120, DBP < 80 as soon as possible to

prevent stroke.

B) Reduction of DBP to 100-105 over 3 hours with maximum fall in BP by 25% over 24

hours is needed to prevent stroke.

C) Heart rate control is acutely needed to prevent worsening cerebral edema. D) Blood pressure should be maintained at current elevated levels.

17 . A 35-year-old healthy man was seen for an annual medical review as part of his company employment policy. Examination reveals oral thrush in an otherwise fit and healthy man. Further tests confirm oral candidiasis. He denies any illness and he is receiving no drugs. Which one of the following is the MOST LIKELY underlying disease?

A) Lymphoma B) Alcoholism

C) HIV infection

D) Chronic renal failure E) Heroin addiction

18 . A 58-year-old chronic alcoholic was admitted with increasing shortness of breath and a distended abdomen. Physical examination revealed evidence for ascites and pitting oedema in both legs. The pulse was 94 bpm, blood pressure 110/60 mmHg and the jugular venous pressure was not raised. The condition LEAST LIKELY to be responsible for this clinical presentation is

A) Alcoholic cardiomyopathy B) Liver cirrhosis

C) Bowel cancer with liver metastasis D) Tuberculous peritonitis E) Portal vein thrombosis

19 . Which one of the following is most characteristic of achondroplasia?

A) One parent is affected in most cases

B) Absence of osteoclasts in the bone marrow is the hallmark of this disorder C) The limbs are short but the trunk height is normal D) Affected individuals are usually sterile E) Intelligence is subnormal

5

—————————————————–Page 5—————————————————–

20 . A 62-year-old man presents to an ER with substernal, crushing chest pain for 30 minutes. He is diaphoretic and short of breath. His only past medical history is that he suffered a GI bleed from peptic ulcer disease 6 months ago and has had no recurrence. Physical examination is notable only for a BP of 160/95. An ECG shows ST segment elevation of 2 mm in leads V1 through V4. The nearest catheterization lab is at a hospital approximately 2 hours away. Which of the following is THE MOST APPROPRIATE NEXT action?

A) He should not be given thrombolytics because he has had a recent GI bleed

B) He should not be given thrombolytics because the catheterization lab is too far away and

he has no contraindications

C) He should not be given thrombolytics because his systolic blood pressure is too high and

places him at risk for hemorrhage

D) He should be referred for emergency catheterization

21. Which of the following oral hypoglycemic drugs can be given safely for patient with chronic renal failure

A) Glyburide B) Nateglinid

C) Repaglinide D) Glipizide

E) Tolazomide

22 . A 62-year-old man with CKD 5 (eGFR 11 mL/min) due to HTN comes to the clinic for routine follow-up with no complaints of nausea, vomiting, fatigue, or poor appetite. An arterio-venous fistula (AVF) was placed 1 year ago when his eGFR was 12 mL/min and the fistula is ready for use. His BP is 134/80 mmHg. He walks 2 miles every day without shortness of breath, or chest pain, or fatigue. Pertinent labs include: Na+ 139 mEq/L, K+ 4.4 mEq/L, HCO3 22 mEq/L, BUN 68 mg/dL, Ca2+ 8.8 mg/dL, phosphate 4.2 mg/dL, and albumin 4.1 g/dL. He expresses hemodialysis (HD) as his choice of renal replacement therapy (RRT). According to the KDIGO guideline, which one of the following is THE MOST appropriate management in this patient?

A) Start HD in 2 weeks at the outpatient dialysis unit B) Start peritoneal dialysis (PD) in 4 weeks

C) Convince for preemptive kidney transplantation

D) Start HD when signs and symptoms of kidney failure are present

E) Suggest no RRT at any time, as he may do well with conservative management

23. One of your 52-year-old female patients with an eGFR of 12 mL/min comes to your office with complaints of fatigue and poor appetite due to metallic taste in the mouth for the last 2 years. She has no chest pain or hyperkalemia. Her BP is 130/80 mmHg. Her serum [HCO3] is 20 mEq/L. During her previous visit 3 months ago, you discussed about the choice of her future renal replacement therapies, including transplantation. After prolonged discussion, she feels that PD is a reasonable choice in view of her daily work in the school. Based on her clinical history, which one of the following choices is MOST appropriate regarding the management of her symptoms and selection of dialysis modality is CORRECT ?

A) Admit to hospital and start HD with a central vein catheter (CVC), and then start PD B) Tell her that she would receive in-center HD once acute HD treatment is over

C) Admit to hospital and place a PD catheter, and then train her for continuous ambulatory

PD with small volume exchanges initially

D) Change her diet and increase her NaHCO3 to 1350 mg every 8 h

E) Tell her that she may not need any renal replacement therapy (either HD or PD) until her

eGFR falls to 7 mL/min

6

—————————————————–Page 6—————————————————–

24. A 55-year-old women is referred for evaluation of hypercalcemia. Three years ago, she had a left mastectomy for breast carcinoma, after which she underwent treatment with radiation and adjuvant chemotherapy because 2 of 12 lymph nodes were found to be positive for carcinoma. She has been clinically well since then, but on a routine postoperative check-up 3 months ago, she was found to have a serum calcium level of 2.75 mmol/L (N:2.2-2.6 mmol/L). On repeat testing, the serum calcium level is 2.9 mmol/L. The best first diagnostic step to identify the cause of this patient’s hypercalcemia is:

A) Serum alkaline phosphatase level B) Serum parathyroid hormone level C) Serum 1,25- dihydroxyvitamin D level D) Serum 25 hydroxyvitamin D level E) Bone Scan.

25. A man drinks 2 L of water to replenish the fluids lost by sweating during a period of exercise. Compared with the situation prior to the period of sweating,

A) His intracellular fluid will be hypertonic B) His extracellular fluid will be hypertonic C) His intracellular fluid volume will be greater D) His extracellular fluid volume will be greater

E) His intracellular and extracellular fluid volumes will be unchanged

26 . A patient with reduced VC, FRC, and RV is found to have a normal pH. A tentative diagnosis of diffuse interstitial fibrosis is made. Which of the following characteristics are consistent with this disease?

A) An increase in lung compliance B) A decrease in respiratory rate C) An increase in the V/Q ratio D) A decrease in PaCO2

E) An increase in the FEV1/FVC ratio

27 . An HIV-positive patient asks if you can tell him the chances of him progressing to symptomatic AIDS. Which one of the following tests would be most useful?

A) CD4 lymphocyte count B) HIV antibody test C) HIV RT PCR D) Neopterin

E) HIV p24 antigen.

28. A 43-year-old male is recovering from an infectious disease and experiences a marked instability in his blood pressure with episodes of spiking of blood pressure. After a series of extensive examinations, it was concluded that this disorder was due to the effects of the infectious agent upon a component of the peripheral nervous system.

Logical sites where an infectious agent could produce such an effect include the

A) Superior ganglia of cranial nerves IX and X B) Geniculate and trigeminal ganglia C) Otic and superior salivatory ganglia D) Carotid sinus and aortic arch E) Carotid and aortic bodies

7

—————————————————–Page 7—————————————————–

29 . A 53-year-old woman returns for re-evaluation of cough of 1 year duration. The cough is occasionally productive, occurs during the day and night, and is triggered by talking, laughing, and cold air. Her pulmonary function tests are normal. The cough has not responded to appropriate therapy for asthma with inhaled corticosteroids and short-acting β -agonists. She denies any history of lung disease, cigarette smoking, atopy, rhinitis, gastroesophageal reflux disease, or obstructive sleep apnea. The physical examination is unrevealing. Further evaluation, including esophageal impedance and manometry, were unremarkable. What is the appropriate next step in this patient’s management? A) High-resolution computed tomography of the chest B) Computed tomography of the sinuses C) Bronchoscopy

D) Echocardiogram

E) Video laryngeal swallowing study

30 . A patient who has an absolute neutrophil count below 500 cells/mm3 for 2 weeks after induction therapy for acute myelogenous leukemia is at increased risk for

A) E. coli bacteremia from the gastrointestinal tract

B) Influenza infection involving the upper respiratory tract C) Varicella-zoster virus reactivation

D) Cytomegalovirus disease involving the lungs E) Cryptococcus neoformans meningoencephalitis

31. What is THE MOST LIKELY cause of neutropenic enterocolitis that developed after receipt of cytotoxic therapy for acute myelogenous leukemia?

A) Cytomegalovirus B) Clostridium difficile C) Norovirus

D) Mixed anaerobic and aerobic bacteria E) Adenovirus

32 . A 63-year-old woman presents with a 6-month history of midsternal chest pressure that occurs both at rest and with exertion. Her only cardiac risk factor is chronic hypertension with medical therapy. She also has osteoarthritis of her knees and is able to walk only short distances. The baseline electrocardiogram shows left ventricular hypertrophy with a strain pattern. An appropriate diagnostic test to evaluate this patient for coronary disease is

A) Treadmill electrocardiographic stress test B) Treadmill echocardiographic stress test C) Supine bicycle nuclear perfusion stress test D) Dobutamine stress echocardiography E) Coronary angiography

33. Which one of the following drugs may be difficult to remove in an overdose setting because of a large volume of distribution (VD)?

A) Procainamide B) Aspirin C) Digoxin

D) Phenobarbital E) Phenytoin

8

—————————————————–Page 8—————————————————–

34. A neurological examination of a 75-year-old male reveals that when the abdominal wall is stroked, the muscles of the abdominal wall of the side of the body stimulated failed to contract. Other neurological tests appeared normal. The likely region affected includes

A) C1–C5 spinal segments B) C6–T1 spinal segments C) T2–T7 spinal segments D) T8–T12 spinal segments E) L1–L5 spinal segments

35 . A 55-year-old woman manager of a regional long-distance telephone office whom you examine for the first time feels well. You do a complete physical examination, which is normal except for a few very small palpable and moveable, nontender nodes in both cervical chains and occasional wheezes in the lungs. However, her laboratory studies show the following hepatitis B virus profile: negative HBsAg, positive anti-HBs, low levels of IgG anti-HBc, negative anti-HBeAg, and positive anti-HBe. The most likely diagnosis is

A) Acute HBV infection, high infectivity B) Late-acute HBV, low infectivity C) Recovered from HBV infection

D) Chronic HBV infection, high infectivity E) Immunization with HBsAg vaccine

36 . A 63-year-old female is hospitalized secondary to markedly decreased vision. She has no history of polydipsia or nocturia. Physical examination finds bilateral sluggish light reflexes and a bitemporal hemianopsia. No papilledema is present, and her urine specific gravity is within normal limits. A CT scan of the head finds a suprasellar mass with calcification. What is the most likely diagnosis for this tumor?

A) Craniopharyngioma B) Germinoma

C) Juvenile pilocytic astrocytoma D) Medulloblastoma E) Meningioma

37 . After receiving incompatible blood, a patient develops a transfusion reaction in the form of back pain, fever, shortness of breath, and hematuria. This type of immunologic reaction is classified as a

A) Systemic anaphylactic reaction

B) Systemic immune complex reaction C) Delayed type hypersensitivity reaction

D) Complement-mediated cytotoxicity reaction E) T cell–mediated cytotoxicity reaction

38 . A male patient is brought to the emergency department (ED) following ingestion of an unknown substance. He is found to have an elevated temperature, hot and flushed skin, dilated pupils, and tachycardia. Of the following, which would most likely cause these findings?

A) Propranolol

B) Methylphenidate C) Prazosin

D) Guanethidine E) Atropine

9

—————————————————–Page 9—————————————————–

39 . A 65-year-old man was diagnosed with lung cancer 6 months ago. Over the last 2 months, he has had worsening severe proximal muscle weakness. He is most likely to have which of the following?

A) Dermatomyositis B) Trichinosis

C) Multiple sclerosis (MS)

D) Progressive multifocal leukoencephalopathy (PML) E) Myasthenia gravis

40 . A 45-year-old woman with long-standing, well-controlled rheumatoid arthritis develops severe pain and swelling in the left elbow over 2 days. She is not sexually active. Arthrocentesis reveals cloudy fluid. Synovial fluid analysis reveals greater than 100, 000 cells/mL; 98% of these are PMNs. What is the most likely organism to cause this scenario?

A) Streptococcus pneumoniae B) Neisseria gonorrhoeae C) Escherichia coli

D) Staphylococcus aureus E) Pseudomonas aeruginosa

41. An obese 50-year-old woman complains of insomnia, daytime sleepiness, and fatigue. During a sleep study she is found to have recurrent episodes of arterial desaturation about 30 events per hour with evidence of obstructive apnea. Which of the following is the treatment of choice for this patient?

A) Nasal continuous positive airway pressure B) Uvulopalatopharyngoplasty C) Hypocaloric diet D) Tracheostomy

E) Oxygen via nasal cannula

42 . An 18-year-old man comes to see you with erythematous plaques over his elbows and knees, which have progressed over the last 2 months. He is concerned about his appearance. The lesions have a fine, silvery scale. Which one of the following treatments is not used for this condition?

A) Corticosteroids.

B) Benzoyl peroxide.

C) Vitamin D derivatives. D) UVB radiation therapy. E) Anti-TNF therapy.

43 . Which one of the following is true of HIV infection?

A) HIV-2 is the most common type worldwide. B) It predominantly depletes CD8 T cells.

C) The risk of HIV transmission from a needle-stick injury is lower than that of hepatitis B

transmission.

D) Persistent generalized lymphadenopathy is an AIDS indicator illness. E) Drug therapy is only started when the CD4 count falls below 200 cells/mL.

10

—————————————————–Page 10—————————————————–

44 . Familial hypercholesterolemia (FH) is a common autosomal dominant disorder resulting from mutations leading to impaired hepatic clearance of low-density lipoprotein (LDL) from the circulation. All of the following statements are true regarding heterozygous FH EXCEPT that

A) It occurs in 1 in 5,000 persons.

B) It is associated with serum low-density lipoprotein cholesterol (LDL-C) two to three times

above the average.

C) It is associated with four to six-fold increased risk of premature coronary heart disease

(CHD).

D) Without treatment, the average age for development of symptomatic CHD is 45 years in

men and 55 years in women.

E) Ninety percent of FH heterozygotes exhibit detectable xanthomas on the extensor

tendons of the hands or on the Achilles tendons by the age of 39.

45 . A 40-year-old man has epistaxis and bruises easily. As a child he had excessive bleeding following tonsillectomy and teeth extractions. His mother and sister had similar symptoms. The most likely diagnosis is?

A) Haemophilia A B) Haemophilia B

C) von Willebrand’s disease

D) Idiopathic thrombocytopenic purpura E) Lupus anticoagulant

46 . A 50-year-old man is evaluated for poorly controlled hypertension. His blood pressure has been elevated for 12 years and remains between 150/105 mm Hg and 170/105 mm Hg despite the use of multiple medications. He also has poor exercise tolerance and fatigue and often falls asleep in the afternoon. Medications are atenolol, 50 mg/dl. On physical examination, blood pressure is 168/110 mm Hg. He is obese and appears plethoric. The remainder of the examination is normal. Glucose (fasting) :102 mg/dL (5.66 mmol/L), Blood urea nitrogen :20 mg/dL (7.14 mmol/L), Creatinine: 1.4 mg/dL (123.79 umol/L),Sodium: 140 meq/L (140 mmol/L), Potassium: 3.9 meq/L (3.9 mmol/L), Bicarbonate: 25 meq/L (5 mmol/L), Cholesterol: 220 mg/dL (5.69 mmol/L), Triglycerides: 190 mg/dL (2.15 mmol/L), High-density lipoprotein cholesterol: 37 mg/dL (0.96 mmol/L)

Which of the following is THE MOST LIKELY cause of this patient’s resistant hypertension?

A) Renovascular hypertension B) Primary hyperaldosteronism C) Pheochromocytoma

D) Type 2 diabetes mellitus E) Sleep apnea syndrome.

47. A 28-year-old woman presents with extreme pallor, palpitations and lack of energy. The full blood count reveals: hemoglobin 4 g/dl, WBC 12 x 109/l, MCV 112 fl, platelet count of 500 x 109/l, reticulocyte count 20%. The finding of depleted iron stores is highly suggestive of which of the following conditions?

A) Sickle cell anemia B) Thalassemia major

C) IgG warm antibody-mediated hemolytic anemia D) Spherocytosis

E) Paroxysmal nocturnal hemoglobinuria

11

—————————————————–Page 11—————————————————–

48. A 30-year-old woman is referred because of recent onset headache and dizziness. Physical examination reveals blue lips and blue fingertips. Urgent blood gases on air show a Pao2, of 12 kPa (10-14 kPa). This patient is probably taking which one of the following medications?

A) Dapsone

B) Sodium valproate C) Phenytoin

D) Zinc sulphate E) Nitrous oxide

49. A 68-year-old woman presents with a six-month history of insidious onset of fatigue, poor appetite and morning stiffness predominantly in the shoulders. Physical examination was unremarkable except for pallor. Laboratory studies reveal normal WBC, hematocrit of 27%, MCV 70 fl, platelets 700 x 10 9 /l and ESR 78 mm/h. Bone marrow aspirate is normal, with

increased iron stores. THE MOST LIKELY diagnosis is

A) Essential thrombocytosis B) Hypothyroidism C) Myelodysplasia

D) Polymyalgia rheumatica E) Polycythemia rubra vera

50. A 22-year-old girl presented because of fatigue and excessive daytime sleepiness, she describes 3 to 4 naps during day time and every time she awake from sleep she feels active and fresh but this doesn’t last long. Her TSH level was normal. Diagnostic polysomnography showed apneal/hypopnea index of 6/hr. The MOST APPROPRIATE NEXT step in the management of this patient is: A) Start nasal CPAP treatment.

B) Order titration of CPAP pressure.

C) Order multiple sleep latency (MSLT) test. D) Advice the patient to lose weight.

E) Advice the patient to use dental device.

51. A 67-year-old woman with a long history of rheumatoid arthritis comes to the clinic for review. Her rheumatoid is poorly controlled and she is receiving regular doses of methotrexate and low dose prednisolone. Most recently she has begun to suffer from increasing pitting edema. On examination her BP is 145/84 mmHg and her pulse is 85. She has pitting edema to the mid shin. Investigations show: Hemoglobin 110 g/L (115- 160); White cell count 8.3 ×10 9 /L (4- 11); Platelets 159 ×10 9 /L (150- 400); Serum sodium 140 mmol/L (135- 146); Serum potassium

4.4 mmol/L (3.5- 5); Creatinine 130 µmol/L (79 – 118); Serum albu min 24 g/L (36- 50); Urine Protein ++.

Which of the following is the most appropriate investigation likely to elucidate the underlying diagnosis?

A) Rectal biopsy B) Renal biopsy

C) Renal ultrasound scan D) Skin biopsy E) Liver biopsy

12

—————————————————–Page 12—————————————————–

52. An 84-year-old man presents with right upper arm pain which he has had for the last few months. The pain is worsening progressively, and wakes him up at night.

He is known to have Paget’s disease involving lumbar spine and pelvis, and is on oral

bisphosphonates for this. There is no history of injury. On examination the shoulder movements are free, and normal. What is THE MOST LIKELY cause of his arm pain?

A) Fracture

B) Osteoarthritis C) Osteonecrosis D) Osteosarcoma E) Paget’s disease

53. A 56-year-old woman presents with a six day history of sharp shooting pain radiating to her right forearm, with paresthesia. On examination, she has sensory loss affecting the centre of her palm and the right hand middle finger. Elbow and finger extension are weak and the triceps jerk is absent. Which cervical spine nerve root is affected?

A) C5 B) C6 C) C7 D) C8 E) T1

54. A 60 year old woman is assessed in the rheumatology clinic for osteoporosis. She is 1 and a half years post-menopausal and has a family history of osteoporosis. What therapy should be commenced?

A) Vitamin D supplements

B) Cyclical estrogen progestogen

C) Calcium and vitamin D supplements D) Etidronate

E) Calcium supplement

55 . A 56-year-old lady has a known ventricular septal defect. Which one of the following clinical signs would most indicate the presence of established pulmonary hypertension?

A) Loud systolic murmur

B) Raised jugular venous pressure C) Single loud second heart sound D) Systolic thrill

E) Displaced apex beat

56. A 38-year-old woman is evaluated while in a stupor state. The patient is slender and nearly comatose. She responds minimally to loud voice and sternal pain. On physical examination, the pulse rate is 110/minute and blood pressure is 115/70 mmHg. An accompanying friend informs you that the patient is a nurse and she has had recent psychiatric problems. Serum glucose level is 14 mg/dL, additional blood is drawn, and the patient is quickly resuscitated with intravenous glucose. A sulfonylureas screen is negative. Laboratory calcium: 9.5 mg/dl – Serum insulin: 45 mU/ml(normal: 5- 20 mU/ml); C-peptide: 0.6 ng/ml(normal 0- 0.2 ng/ml); Proinsulin: 0.6 ng/ml(normal 0-0.2 ng/ml) Which of the following is THE BEST explanation of these finding?

A) Solitary insulin-producing pancreatic islet cell tumor B) Surreptitious insulin use

C) Surreptitious metformin use

D) Multiple endocrine neoplasia type 1 E) Multiple endocrine neoplasia type 2A

13

—————————————————–Page 13—————————————————–

57. A 42-year-old man has a history of splenectomy following trauma at the age of 20. He received appropriate vaccinations immediately after his trauma but has not had any medical care for more than 10 years. He is homeless. He presents to the emergency department with a fever of 102.3°F (39.1°C), blood pressure (BP) of 7 0/40 mmHg, heart rate (HR) of 130 bpm, respiratory rate (RR) of 30 breaths/min, and oxygen saturation (SaO2) of 95% on room air. What is the best initial antibiotic therapy for this patient?

A) Ceftriaxone and vancomycin

B) Ceftriaxone, ampicillin, and vancomycin

C) Ceftriaxone, vancomycin, and amphotericin B D) Clindamycin, gentamicin, and vancomycin E) Clindamycin and quinine

58 . You are determining whether or not to use a rapid streptococcal antigen test to screen for streptococcal pharyngitis. You find that 2% of people with strep throat actually test negative using this test. Which of the following statements best describes this situation?

A) The sensitivity of the test is 2%. B) The specificity of the test is 98%.

C) The test has a 2% false-negative rate. D) The test has a 2% false-positive rate.

E) The test has a positive predictive value of 98%.

59 . A 55-year-old type II diabetic male nonsmoker is consulting you electively for the advisability of taking up a controlled exercise program. He has no history of chest pain or heart disease incidents or symptoms of heart failure. However, a routine ECG shows a QS configuration in leads II, III, and AVF (deep Q waves; no R wave); there is no ST elevation or depression. Each of the following statements is true about this situation EXCEPT which one?

A) The patient has had a posterior MI of indeterminate age. B) “Silent” MIs carry a worse prognosis than symptomatic MIs. C) “Silent” MI is more likely to occur in diabetics.

D) During the acute phase of this past MI, the patient was likely to exhibit a sinus

bradycardia.

E) During the acute phase of this MI, the patient’s symptoms were easily confused with an

upper gastrointestinal syndrome.

60. A 28-year-old female presents to her internist with a 2-day history of low-grade fever and lower abdominal pain. She denies nausea, vomiting, or diarrhea. On physical examination, there is temperature of 38.3ºC (100.9ºF) and bilateral lower quadrant tenderness, without point or rebound tenderness. Bowel sounds are normal. On pelvic examination, an exudate is present and there is tenderness on motion of the cervix. Her white blood cell count is 15,000/µL and urinalysis shows no red or white blood cells. Serum β -hCG is undetectable. Which of the following is the best next step in management? A) Treatment with ceftriaxone and doxycycline B) Endometrial biopsy C) Surgical exploration D) Dilation and curettage E) Aztreonam

14

—————————————————–Page 14—————————————————–

61. 57-year-old male with symptomatic osteoarthritis of the knees and hips has had chest pains at odd times, not necessarily related to exercise, and sometimes associated with emotional stress. His resting pulse rate is 52, sinus rhythm. An ECG shows Wenckebach phenomenon. You decide to do a cardiac stress test. Which of the following would be the most acceptable option?

A) Phased treadmill exercise stress test

B) Thallium sestamibi scintigraphy (scan for ischemia before and after exercise) C) Adenosine chemical stress test

D) Dobutamine, radionuclide angiography E) Stress echo test

62. A 25-year-old woman is admitted with fever and hypotension. She has a 3-day history of feeling feverish. She has no history of chronic disease, but she uses tampons for heavy menses. She is acutely ill and, on physical examination, found to have a diffuse erythematous rash extending to palms and soles. She is confused. Initial blood tests are as follows: White blood cell count: 22,000/µL; Na + : 125 mEq/L; K + : 3.0 mEq/L; Ca ++ : 8.0 mEq/mL; Activ ated

partial thromboplastin time (PTT): 65 (normal 21 to 36s) Prothrombin time (PT): 12s (normal 12

to15s); Aspartate aminotransferase: 240 U/L (normal 40); Creatinine: 3.0 mg/dL; Antinuclear

antibodies: negative; Anti-DNA antibodies: negative; Serologic tests for RMSF; leptospirosis, measles: negative. Which of the following best describes the pathophysiology of the disease process?

A) Acute bacteremia

B) Toxin-mediated inflammatory response syndrome C) Exacerbation of connective tissue disease D) Tick-borne rickettsial disease E) Allergic reaction

63 . A 50-year-old man experienced the sudden onset of vertigo. Examination revealed dysarthria, difficulty in swallowing, a left Horner’s syndrome, left palatal weakness, and loss of pain sensibility over the left face and the right limbs and trunk. He had coarse ataxia and in- coordination of his left arm. The patient was alert and anxious trying to communicate with his dysarthria. He did complain of a mild occipital headache. The cause of the responsible lesion is most likely:

A) Subarachnoid hemorrhage

B) Infarction, middle cerebral artery C) Infarction, vertebral artery

D) Infarction, posterior inferior cerebellar artery E) Pontine hemorrhage

64. A 30-year-old male with sickle cell anemia is admitted with cough, rusty sputum, and a single shaking chill. Physical examination reveals increased tactile fremitus and bronchial breath sounds in the left posterior chest. The patient is able to expectorate a purulent sample. Which of the following best describes the role of sputum Gram stain and culture?

A) Sputum Gram stain and culture lack the sensitivity and specificity to be of value in this

setting.

B) If the sample is a good one, sputum culture is useful in determining the antibiotic

sensitivity pattern of the organism, particularly Streptococcus pneumoniae.

C) Empirical use of antibiotics for pneumonia has made specific diagnosis unnecessary. D) There is no characteristic Gram stain in a patient with pneumococcal pneumonia. E) Gram-positive cocci in clusters suggest pneumococcal infection.

15

—————————————————–Page 15—————————————————–

65 . A rheumatology inpatient has recently been diagnosed with granulomatosis with polyangiitis (Wegener’s granulomatosis). First-line treatment would be which one of the following?

A) NSAIDs B) Steroids

C) Oral DMARDs

D) Cyclophosphamide E) Plasma exchange

66. A 50-year-old man presents with a 6-week history of generalized joint pain and fatigue associated with a persistent rash on his legs. He also complains of dryness of his mouth and eyes. His only past history was of diabetes mellitus diagnosed 3 years ago, which he manages with diet alone and of a motorcycle accident 15 years ago in which he sustained a ruptured spleen, which was removed surgically. On examination he had no swelling or limitation of joint movement but there was tenderness of peripheral joints on palpation. There was a purple raised rash on his lower limbs and a small ulcer on the left shin. Neurological examination revealed decreased sensation in both feet in a stocking distribution. His investigations revealed normal CBC, raised creatinine at 198 µmol/L, AST 135 U/L, ALT 123 U/L, ALP 234 U/L, bilirubin 15 µmol/L, ANA was negative but rheumatoid factor was positive in high titer. His urine dipstick showed blood 2+ and protein 3+. THE MOST LIKELY diagnosis is :

A) Sjogren’s syndrome with vasculitis B) SLE with vasculitis

C) Diabetic complications D) Rheumatoid arthritis

E) Cryoglobulinemia with hepatitis C infection

67 . A 63-year-old man who is known to have tuberculosis is referred with shortness of breath and increasing ankle swelling. On examination a raised jugular venous pulse (JVP) and soft heart sounds are noted and he has pitting edema below the knee. There is also an inspiratory increase in venous pressure and a steep y descent in the JVP. Which one of the following is THE MOST LIKELY diagnosis?

A) Constrictive pericarditis

B) Active pulmonary tuberculosis C) Cor pulmonale

D) Cardiac tamponade

E) Right-sided heart failure

68 . Which of the following factors is least useful in assessing patients with a poor prognosis in community acquired pneumonia?

A) Mini-mental score of 6/10 B) Urea of 11.4 mmol/l

C) C-reactive protein of 154 D) Respiratory rate of 30 E) Aged 75 years old

69. Which of the following is used to establish the definitive diagnosis of relapsing fever?

A) Serologic tests B) Blood culture C) Urine culture

D) Demonstration of the organism in peripheral blood E) Animal inoculation

16

—————————————————–Page 16—————————————————–

70. A 67-year-old Cambodian woman sustained a deep laceration on her leg while gardening. The wound was visibly contaminated with soil. She immigrated to Qatar 5 years ago and lives with her son and family. She is not aware of having any vaccinations as a child. She takes no medications and has no medication allergies. Which of the following should you recommend for tetanus prophylaxis?

A) Human tetanus immunoglobulin (HTIG) B) Primary series of tetanus toxoid (T)

C) Primary series of tetanus and diphtheria toxoids (Td)

D) Primary series of diphtheria and tetanus toxoids and pertussis (DTaP) vaccine E) HTIG and primary series of Td

71 . A 65-year-old man reports progressive unintentional weight loss of 25 pounds (10% of body weight) during the past 12 months. He reports no dysphagia, nausea, vomiting, or jaundice. His appetite remains good. He reports loose stools and excessive, foul-smelling flatus. He has a history of diabetes mellitus, for which he takes metformin and insulin. He has a prior history of heavy ethanol use but has been abstinent for several years. A physical examination is unrevealing, including a nontender abdomen and no organomegaly or abnormal masses. Initial blood work including a compete blood count, electrolytes, and liver chemistries is normal. Which of the following is THE MOST LIKELY cause of the patient’s weight loss?

A) Metformin

B) Chronic pancreatitis C) Colon cancer

D) Diabetic gastroparesis E) Pancreatic cancer

72. A 40-year-old woman with a history of atopy, rhinitis, and gastroesophageal reflux disease (GERD) presents with wheezing of 3 months duration. The wheezing occurs primarily during the day, is triggered by cold air and exercise, and has not responded to a 6-week course of an inhaled corticosteroid and short-acting β -agonist. She denies chest tightness, chest pain, or a history of lung or cardiovascular disease. Her history is notable for allergies to dust, multiple weeds, grasses, and animal dander. She has persistent rhinitis for which she performs nasal saline rinses and uses intranasal corticosteroids daily. She also takes a proton pump inhibitor once daily for treatment of GERD, and these symptoms are well controlled. Her physical examination reveals normal vital signs and erythema of the upper airway without nasal polyps. The remainder of her physical examination is normal without wheezing. Pulmonary function tests, chest radiograph, brain natriuretic peptide, and methacholine challenge testing are normal. What is the next step in this patient’s evaluation?

A) Sinus computed tomography

B) High-resolution chest computed tomography C) Direct laryngoscopy after exercise D) Echocardiogram

E) Bronchoscopy with biopsy

73. Which of the following medications used to treat pituitary tumor syndromes acts by blocking a hormone receptor?

A) Pasireotide B) Cabergoline C) Ketoconazole D) Mifepristone E) Bromocriptine

17

—————————————————–Page 17—————————————————–

74. A 65-year-old woman presents with complaints of increasing pain in her fingers and hands. She states that she has morning stiffness in those regions that tends to resolve within 10 to 15 minutes after start of activity.

There are no other rheumatic or systemic symptoms and physical examination of the hands is completely normal. Which of the following is the most likely clinical diagnosis?

A) Osteoarthritis

B) Rheumatoid arthritis C) Fibromyalgia

D) Scleroderma/systemic sclerosis

E) Systemic lupus erythematosus (SLE)

75 . Magnetic resonance imaging (MRI) on a 73-year-old woman experiencing dizziness reveals an incidental 3-mm lesion in her pituitary that is compatible with a pituitary adenoma. She has otherwise been well. Testing for hormone over-secretion is negative. What should be THE NEXT STEP in management?

A) Refer her to an experienced neurosurgeon.

B) Repeat the MRI in 1 year to look for size change.

C) Refer her to ophthalmology for a visual field examination. D) Begin bromocriptine treatment.

E) Refer her for a preoperative cardiac stress test.

76. A 65-year-old woman presents with 6 months of progressive lower extremity pain and describes difficulty ascending stairs because of pain and weakness. She also brings a recent outside DXA bone density test, which shows total hip T and Z scores of −4.5 and −2.5,

respectively. She does not have a history of fragility fractures. Past medical history is notable for hypertension and long-standing irritable bowel syndrome. She takes 600 mg of calcium and 400 IU of vitamin D daily, but no other medications. Family history is negative for osteoporosis or parental hip fracture. She does not smoke or drink alcohol. Physical examination is notable for tenderness to palpation over the mid-tibia bilaterally. She also has a wide-based, nonantalgic gait. What is THE NEXT BEST choice for her skeletal management?

A) Double her calcium and vitamin D daily intake. B) Measure serum 25(OH)D level. C) Start an oral bisphosphonate.

D) Measure fasting serum C telopeptide level (CTx) E) Start teriparatide therapy.

77 . A 35-year-old woman is brought to the emergency department with a history of acute-onset nausea, headaches, and stupor after a protein-rich meal. She has a history of similar events in the past. Her brother died as an infant of an encephalopathic crisis. The plasma ammonium level on admission is 400 µ mol/L (reference: <40 µ mol/L). What is THE MOST APPROPRIATE initial treatment?

A) Enzyme replacement therapy B) Liver transplantation C) Intravenous fluids only

D) Alternative pathway therapy with intravenous sodium benzoate or sodium phenylbutyrate E) Hydroxycobalamin IM

18

—————————————————–Page 18—————————————————–

78 . Management of a non−ST elevation acute coronary syndrome (ACS) in women should differ from men in which of the following ways?

A) Medical management is preferred over revascularization in women but not in men. B) Noninvasive testing is not recommended in women.

C) Aspirin is not indicated for emergent treatment of ACS because of bleeding risk. D) β -Blockers should be avoided because of increased risk for congestive heart failure. E) Lipid management goals are low-density lipoprotein cholesterol levels of less than 120

mg/dL.

79. A 38-year-old woman has had type 1 diabetes mellitus since the age of 12 years. She has maintained excellent control (HbA1c 6.0%) with a basal/ bolus injection regimen. She tests her glucose level four or five times a day, and review of her meter download shows many glucose levels in the 30s and 40s. However, the patient is unconcerned because she has no symptoms at these times. On questioning, she admits to recently “spacing out” while driving, which led to a minor traffic accident. Regarding the etiology and treatment of hypoglycemia in this patient:

A) She has adapted to low blood glucose concentration and no change in treatment is

required.

B) She has developed hypoglycemia unawareness and her target HbA1c should be

increased.

C) Strict avoidance of hypoglycemia is of little benefit in reversing hypoglycemia-associated

autonomic failure.

D) An excessive counter-regulatory hormone response to hypoglycemia may contribute to

her lack of symptoms.

E) Treatment with β -blocker should be considered.

80 . A 74-year-old man with a history of hypertension and hypercholesterolemia is hospitalized after presenting with a 3-month history of progressive fatigue and dyspnea on exertion. His weight on admission is 164 lbs, his pulse is 44 bpm, and lab tests show CPK 528 U/L, TSH 65 mU/L, free T4 0.2 ng/dL, and T4 2.3 µ g/dL. A pharmacologic nuclear stress test reveals findings consistent with diffuse myocardial ischemia. Subsequent coronary angiography reveals diffuse three-vessel disease that is not amenable to percutaneous stenting. A consulting cardiologist has recommended that he undergo coronary artery bypass surgery. What would you recommend?

A) Start levothyroxine at a dose of 125 µ g daily.

B) Administer a 60- µ g intravenous dose of levothyroxine daily. C) Check antithyroid peroxidase and antithyroglobulin antibodies. D) Start levothyroxine at a dose of 12.5 µ g daily.

81 . A 65-year-old man reports symptoms of fatigue and loss of appetite. Physical examination demonstrates jaundice and an abdominal mass. CT scan of the abdomen reveals a mass at the head of the pancreas, with surrounding nodal involvement. THE MOST APPROPRIATE next step in this patient’s care is:

A) Examination of serum biomarker studies including CEA and α -fetoprotein B) Image-guided biopsy of the abdominal mass

C) Initiation of combination chemotherapy for advanced pancreatic cancer D) Referral for surgical resection and subsequent pathologic staging

19

—————————————————–Page 19—————————————————–

82 . CD4+ T cells recognize antigenic peptides presented on which of the following?

A) Epithelial cells

B) Major histocompatibility complex (MHC) class I molecules C) MHC class II molecules D) Costimulatory molecules E) CD80

83 . Which of the following host factors should be considered in selecting an antibiotic regimen for treatment of a patient?

A) Prior history of allergies

B) Whether the patient is currently pregnant or breast-feeding C) Age and sex of the patient D) Options A and B

E) Options A, B, and C

84 . A patient with type 1 diabetes mellitus, on hemodialysis, underwent a deceased donor combined renal-pancreas organ transplantation. The pancreatic duct was drained through the recipient’s urinary bladder. The function of the transplanted kidney and pancreas was excellent. Which of the following compensations would occur for the post-transplantation acid- base disorder?

A) Respiratory hypoventilation

B) Respiratory hyperventilation and increased renal ammoniagenesis C) Metabolic alkalosis due to the severe urinary potassium wasting D) Renal chloride losses to match pancreatic bicarbonate loss E) Initial HCO3 − losses followed by paradoxical aciduria

85 . Recommendations for the prevention of central line–associated blood stream infections (CLABSIs) include all of the following EXCEPT

A) Use of maximal barrier precautions during catheter insertion (e.g., use of a cap, mask,

sterile gown, sterile gloves, and sterile full-body drape)

B) Routine replacement of central venous catheters every 7 days

C) Preparation of the catheter insertion site with an antiseptic agent (e.g., >0.5%

chlorhexidine preparation with alcohol) before catheter insertion

D) Daily review of catheter necessity with immediate removal of catheters that are no longer

necessary

E) Scrubbing of the access port with an antiseptic before accessing the Port

86 . An 83-year-old woman was admitted to the acute medical unit with a three-week history of increasing breathlessness. She had a past medical history of myocardial infarction two years ago and a single coronary stent. Clinical examination revealed dullness to percussion at both bases. Temperature was 36.6°C; pulse was 60 b eats per minute. Blood pressure was 130/80 mmHg. Respiratory rate was 18 breaths per minute and oxygen saturation 94% on air. Full blood count, urea and electrolytes, and liver function tests were all normal. Chest X-ray: bilateral basal effusions with the right slightly larger than the left. How should she be managed?

A) CT chest with contrast timed for pleural enhancement B) Blind diagnostic aspiration of the right pleural effusion C) Empirical diuretics and arrange an echocardiogram

D) Ultrasound guided diagnostic aspiration of left and right pleural effusions E) Ultrasound of the pleural cavities to look for abnormalities

20

—————————————————–Page 20—————————————————–

87 . Integrity of all of the following structures can be monitored by sensory-evoked potentials EXCEPT the

A) Dorsal columns B) Cerebellum C) Thalamus

D) Vestibular-cochlear nerve E) Inferior colliculus

88 . A 15-year-old male with a history of myotonic dystrophy is scheduled for tonsillectomy and adenoidectomy under general anaesthesia. Considerations regarding her anaesthesia management include

A) Postoperative hyperthermia may precipitate a myotonic response B) Myotonia can be relieved by non-depolarizing Relaxants C) Mechanical ventilation is contraindicated

D) Myotonia can be precipitated by succinylcholine E) Neostigmine prevents myotonia

89 . A 55-year-old male with type 2 diabetes mellitus, hypertension, and hyperlipidemia presents with fever, hypotension, and decreased urine output. He is admitted to the intensive care unit with sepsis due to pyelonephritis. In addition to broad-spectrum antibiotics, the patient has received one liter of fluid resuscitation with 0.9 % saline. Two hours after presentation, the blood pressure is currently 73/42 mmHg. Temperature is 36.8 °C (98.2 °F), pulse rate is 120 beats/ minute and respiratory rate is 24 breaths/ minute. Hematocrit is 29 %, blood sugar is 257 mg/dL, and serum lactate is 3.0 mg/dL. Which of the following is the most likely to improve mortality in this particular patient?

A) Continued fl uid resuscitation with colloid added B) Intensive insulin therapy to maintain euglycemia C) Vasopressor therapy D) Bicarbonate infusion

E) Transfusion with packed red blood cell.

90. A 26-year-old woman with recurrent urinary tract infection is admitted for acute pyelonephritis. Urine culture shows E. coli, which is sensitive only to gentamicin. Her renal function is normal. After a loading dose, she is maintained on therapeutic doses of gentamicin. Ten days later, she develops hypokalemia, metabolic alkalosis, hypomagnesemia, and hypocalcemia with normal blood pressure. Which one of the following syndromes has been described with the use of gentamicin?

A) Gitelman syndrome

B) Bartter syndrome type V C) Liddle syndrome D) Gordon syndrome E) Laboratory error

91. Both in hospitalized and clinic patients, hypophosphatemia is a common electrolyte disorder. Which one of the following drugs does NOT cause hypophosphatemia?

A) Imatinib

B) Tenofovir

C) Corticosteroids D) Glucose

E) Bisphosphonates

21

—————————————————–Page 21—————————————————–

92 . A 47-year-old man is brought into the emergency department by ambulance after a motor vehicle accident. He is conscious, alert and, besides the lacerations on his scalp, he seems to be otherwise well. His neck is stabilized by a brace and he asks you to remove it. You assess his neurologic exam to be non-focal, but you obtain cervical neck films anyway. No cervical spine abnormalities are noted and you remove the cervical collar. Suddenly, the patient’s pulse rises to 130/min, his blood pressure drops to 80/50 mm Hg, and his respiratory rate increases to 35/min. His physical examination reveals jugular venous distension, peripheral and perioral cyanosis, absent breath sounds on the left side of his chest, a tachycardic but otherwise normal cardiac examination, and a tender abdomen in the left upper quadrant. THE MOST APPROPRIATE initial step is to

A) Insert a needle thoracostomy into the second intercostal space B) Insert a tube thoracostomy into the pleural space C) Intubate the patient for hypoxemic respiratory failure D) Order a portable chest x-ray

E) Perform a diagnostic peritoneal lavage for a splenic rupture

93 . A 15-year-old boy is brought to the office by his father for a physical examination before summer camp. Since baseball season is in full swing, and he is the pitcher on the varsity team, he comes to the office in uniform to be able to go straight to the game after the appointment. He talks incessantly about the game and his father chimes in every so often to show his support for his son. During the examination, the father talks non-stop about getting his son a full baseball scholarship to an ivy league college. He has been in touch with all of the coaches at the schools. You are surprised to see how muscular the boy has become since his physical examination the previous year. You inquire about weight-lifting, and he says that he works-out in their basement daily. His father bought him a full weight set for his birthday. You suspect that he is taking anabolic steroids. During the evaluation, particular attention should be given to

A) Funduscopic examination B) Neurologic reflexes

C) Respiratory examination D) Testicular size

E) Thyroid examination

94 . A 65-year-old woman was admitted to the intensive care unit for management of septic shock associated with an infected hemodialysis catheter. She was initially intubated on hospital day 1 with the acute respiratory distress syndrome. She had slowly been improving such that her fraction of inspired oxygen (FiO2) had been weaned to 0.40, and she was no longer febrile or requiring vasopressors. On hospital day 7, however, she develops a new fever to 39.4°C (102.9°F) with increased thick yellow-green sputum in her endotracheal tube. You suspect the patient has ventilator-associated pneumonia (VAP). Which of the following makes THE MOST DEFINITIVE DIAGNOSIS of VAP in this patient?

A) An endotracheal aspirate yielding a new organism typical of VAP B) The presence of a new infiltrate on chest radiograph

C) Quantitative cultures from an endotracheal aspirate yielding more than 106 organisms

typical of VAP

D) Quantitative culture from a protected brush specimen yielding more than 103 organisms

typical of VAP

E) There is no single set of criteria that is reliably diagnostic of pneumonia in a ventilated

patient.

22

—————————————————–Page 22—————————————————–

95 . Which of the following acute kidney injury patients is most likely to have evidence of bilateral hydronephrosis on ultrasound evaluation of the kidneys?

A) A 19-year-old man with purpura fulminans associated with gonococcal sepsis B) A 37-year-old woman undergoing chemotherapy and radiation for advanced cervical

cancer

C) A 48-year-old man with chronic renal insufficiency due to hypertension that received

iodinated contrast for an abdominal angiogram

D) A 53-year-old man with Escherichia coli 0157:H7 associated thrombotic

thrombocytopenic purpura

E) An 85-year-old woman who resides in a nursing home with pyelonephritis and sepsis

96 . All of the following statements regarding lung abscesses are true EXCEPT

A) Lemierre syndrome is a lung abscess due to septic thrombophlebitis originating in the

pharynx.

B) Lung abscesses are typically characterized by a >2-cm single dominant cavity. C) Primary lung abscesses are often principally caused by anaerobic bacteria. D) Primary lung abscesses typically are related to oropharyngeal aspiration.

E) Radiographically, primary lung abscesses most commonly involve the middle lobe and

the lingula.

97 . Which of the following GI disorders is characterized by increased absorption from the GI tract into the portal circulation?

A) Celiac disease B) Crohn disease

C) Hemochromatosis D) Pernicious anemia E) Whipple disease

98 . Which of the following patients should receive antibiotic prophylaxis prior to prevent infective endocarditis?

A) A 23-year-old woman with known mitral valve prolapse having a tooth cavity filled B) A 24-year-old woman who had an atrial septal defect completely corrected 22 years ago

who is undergoing elective cystoscopy for painless hematuria

C) A 30-year-old man with a history of intravenous drug use and prior endocarditis

undergoing gingival surgery

D) A 45-year-old man who received a prosthetic mitral valve 5 years ago undergoing

routine dental cleaning

E) A 63-year-old woman who received a prosthetic aortic valve 2 years ago undergoing

screening colonoscopy

99 . A 24-year-old woman is seen in follow-up 12 months after an allogeneic stem cell transplantation for acute myeloid leukemia. She is doing well without evidence of recurrent disease but has had manifestations of chronic graft-versus-host disease. She should be administered all of the following vaccines EXCEPT :

A) Diphtheria-tetanus B) Influenza

C) Measles, mumps, and rubella D) Poliomyelitis via injection

E) 23-Valent pneumococcal polysaccharide

23

—————————————————–Page 23—————————————————–

100. Which of the following autoantibodies is least likely to be present in a patient with systemic lupus erythematosus?

A) Anti-dsDNA

B) Antinuclear antibodies C) Anti-La (SS-B) D) Antiphospholipid E) Antierythrocyte

101 : What is the leading cause of death in patients with chronic kidney disease?

A) Cardiovascular disease B) Hyperkalemia C) Infection

D) Malignancy E) Uremia

102 . Which of the following tissue injury biomarkers has been shown to predict the onset of acute kidney injury after an ischemic or hypotensive event?

A) Blood urea nitrogen (BUN) B) Interleukin-18

C) Kidney injury molecule-1 (KIM-1)

D) Neutrophil gelatinase-associated lipocalin (NGAL) E) None of the above

103 . A 45-year-old woman is admitted to the emergency department after a first episode of witnessed generalized tonic-clonic seizure. She is administered lorazepam 2 mg with cessation of seizure activity. All of the following are likely possible causes of her seizure EXCEPT:

A) Alcohol withdrawal B) Autoantibodies C) Brain tumor

D) Genetic disorder E) Hyperglycemia

104. A 24-year-old third-year medical student is brought to the emergency department from the operating room after she began to have trouble breathing after she put on her gown, latex gloves, and mask. This was going to be her first time “scrubbing in” to a case on the first day of her surgery rotation. Another medical student who was “scrubbed in” to the case tells you that the she was grasping at her throat and attempting to rip off her gown. She is now on a stretcher, gasping for air. She is brought into a room and all of her clothes are taken off, revealing red hands and an eruption of well-circumscribed, erythematous, raised lesions covering her entire body. Her blood pressure is 70/50 mm Hg, pulse is 110/min, and respirations are 45/min. She has marked laryngeal edema and audible wheezes. The other student says that this is the second day of their third year, and that the only other patient contact that they have had was during the first year when they went as a group to evaluate a patient in respiratory isolation. At this time THE MOST CORRECT STATEMENT about her condition is:

A) Her rash is unrelated to her respiratory symptom

B) Her symptoms are caused by the anxiety and stress from the first surgical experience C) Her symptoms are due to an infectious pathogen

D) Her symptoms are the result of bioactive mediators released when exposed to an

antigen

E) She should be given indomethacin immediately to reduce the respiratory symptoms

24

—————————————————–Page 24—————————————————–

105. A 56-year-old man with glioblastoma multiforme in the right parietal lobe experiences his first generalized tonic-clonic seizure. What is the best course of action for this patient?

A) Initiate therapy with ethosuximide B) Initiate therapy with lamotrigine C) Initiate therapy with phenytoin

D) Observe for additional seizures and initiate therapy only if additional seizures occur E) Refer for electroencephalogram (EEG) and treat only if an epileptogenic focus is

identified

106 . A 40 year old woman presented with 5 months history of dyspnea, lately she is unable to walk along distances. During this period of time she was complaining from low grade fever, drenching night sweats, cough, and weight loss of 15 kgs. The leukocyte count is 13,000, serum eosinophil count is 22%. The ESR is 100 mm/hr. Serum IgE level is normal. Chest radiographs demonstrate peripheral infiltrates in the outer two thirds of both lung fields. The FEV1 is 1.3L and FVC is 1.5 L. All of the following statements regarding the management of this case are true EXCEPT

A) BAL typically will show increased CD4/CD8 ratio.

B) Spontaneous remission can be seen in small Percentage of patients. C) CT scan may shows mediastinal lymphadenopathy

D) Relapse of symptoms and chest radiograph Abnormalities occurs if treatment

discontinue before 6 months of treatment.

E) Prednisolone results in dramatic resolution of symptoms within 24 to 48 h.

107 . A patient with Crohn’s disease who has had multiple resections is brought to the emergency department by his family with the chief symptoms of confusion and bizarre behaviour. On physical examination, he has nystagmus and ataxia. Laboratory studies reveal a normal blood ethyl alcohol level and a normal toxin and drug screen results. There is a metabolic acidosis. Lactate levels are normal. The patient’s diagnosis will be confirmed by which of the following blood tests?

A) Serum ammonia level B) Blood copper level C) Serum magnesium D) Vitamin B 12 level E) D -Lactate level

108 . A 73-year-old man has had three episodes of visual loss in the right eye. The episodes last 20 to 30 minutes and resolve completely. He describes the sensation as like a window shade being pulled down in front of the eye. He has a history of hypertension and tobacco use. He denies dyspnea, chest pain, palpitations, or unilateral weakness or numbness. On examination the patient appears healthy; his vital signs are normal and the neurological examination is unremarkable. An ECG shows normal sinus rhythm without evidence of ischemia or

hypertrophy. Initial laboratory studies are normal. Both non-contrast CT scan of the head and MR scan of the brain are normal. What is THE BEST NEXT STEP in this patient’s management?

A) Admit the patient to the hospital for intravenous unfractionated heparin. B) Obtain an echocardiogram.

C) Check for antiphospholipid antibodies and homocysteine levels. D) Order a carotid duplex ultrasonogram and begin antiplatelet therapy. E) Begin lamotrigine for probable non-convulsive seizure.

25

—————————————————–Page 25—————————————————–

109 . A phlebotomist sustains a needle stick injury whilst taking blood from a HIV-positive patient. Which one of the following has been shown to most reduce the risk of HIV transmission?

A) Thorough immediate washing of the injury site with water B) Oral lamivudine therapy for 1 month C) Oral zidovudine therapy for 1 month D) Oral ritonavir therapy for 1 month E) Oral triple therapy for 1 month

110 . A 60-year-old woman complains of dry mouth and a gritty sensation in her eyes. She states it is sometimes difficult to speak for more than a few minutes. There is no history of diabetes mellitus or neurologic disease. The patient is on no medications. On examination, the buccal mucosa appears dry and the salivary glands are enlarged bilaterally. Which of the following best describes the pathophysiology of the condition?

A) Previous exposure to group A streptococcal organisms have stimulated an autoimmune

response that leads to cross-reactivity between host and organism with tissue destruction and reduced tear and saliva production.

B) T cells infiltrate exocrine glands and B cells become hyper-reactive. Auto-antibodies ensue including anti-Ro/SSA and anti-La/SSB. Both pro- and anti-apoptotic messages are sent to ductal and acinar epithelial cells.

C) Activated T cells and monocytes accumulate in the skin leading to induration for unknown reasons. This infiltration leads to structural abnormalities in various tissues and organs hence a reduction in normal functioning. Anti-topoisomerase-I and anti- centromere autoantibodies are commonly present.

D) Immune complexes form and deposit in vessel walls. Vasoactive amines including histamine, bradykinin, and leukotrienes are released, and vessel permeability is increased. Complement activation occurs and mononuclear cells are attracted causing infiltration and decreased gland function.

E) Necrotizing vasculitis of small arteries and veins leads to granuloma formation and

decreased exocrine function of salivary and lacrimal glands.

111. A 75-year-old lady presents with persistent diarrhoea and weight loss. Twenty years previously she had radiotherapy for carcinoma of the ovary. Thirty years ago she lived in Hong Kong for 18 months. Her father died of cancer of the colon aged 62 years. Hb 9.4 g/dL (11.5-16 g/dL); WCC 9.6 X10 9 /L (4-11X10 9 /L); Platelets 234 X 10 9 /L (150-400X10 9 /L); B12 153 pmol/L

(160-900pmol/L); Red cell folate 21 nmol/L (130-630 nmol/L); Ferritin 5 ug/L (20-250 ug/L); Erythrocyte sedimentation rate (ESR) 36 mm/hour . Which of the following is the most likely diagnosis?

A) Radiation enteritis

B) Carcinoma of the colon C) Crohn’s disease D) Tropical sprue

E) Acquired lactose intolerance

112. Which of the following IS MOST LIKELY to be associated with the development of gynaecomastia?

A) Congenital adrenal hyperplasia B) Prolactinoma

C) Hypopituitarism D) Hypothyroidism E) Seminoma

26

—————————————————–Page 26—————————————————–

113. A 30-year-old female with systemic lupus erythematosus is recovering from a fracture of the right femur and right radius following a motor vehicle accident. She has been in the hospital for 5 days. She has a temperature spike of 39.0 °C (102.2 °F). Blood cultures are drawn which grow yeast, species to be identified. She does not appear ill or toxic. No obvious source of infection is found. The patient has an indwelling central catheter. In addition to changing the patient’s central line, which of the following do you recommend?

A) Continued observation.

B) Computed tomography of the chest and abdomen. C) Start fluconazole.

D) Start liposomal amphotericin B.

E) Repeat blood cultures and treat if positive.

114 . A 26-year-old woman presents to the emergency department with shortness of breath as her primary complaint, which has been progressively increasing for several days since starting her menstrual period. She has also been experiencing increasing weakness during the past week as well. She notes a worsening of her symptoms at the end of the day, and she has noticed weakness while brushing her hair. Occasionally she reports blurry vision or difficulty with reading. On physical exam, no specific weakness is noted. She seems fatigued in general and has a depressed affect. Pulmonary exam is normal. All labs are within normal range.

Which of the following neuromuscular disorders is most likely the cause of this patient’s symptoms?

A) Guillain–Barré syndrome

B) Bilateral diaphragmatic paralysis C) Myasthenia gravis

D) Duchenne muscular dystrophy

E) Amyotrophic lateral sclerosis (ALS)

115. You are called to see a patient on the floor for acute obtundation. She has been admitted for chronic abdominal pain and possible pancreatitis. She has been in the hospital for 6 h and has received several doses of narcotics. Her last dose was 4 mg of hydromorphone 15 min ago. She has been on hydromorphone every 4 h. After you assess the patient, you administer a dose of naloxone. The patient has an immediate improvement of her symptoms. She is alert and oriented. One hour later, you are called again to see the patient, who has become somnolent again. The most likely cause of patient’s worsening mental status is:

A) Worsening CO 2 retention

B) Diminishing effects of naloxone C) Further narcotic use D) Sepsis

E) Delirium secondary to pancreatitis

116 . A 40-year-old woman comes to your office with complaints of gradually worsening memory. Her husband states that she has become irritable and “difficult to live with” over the past 9-12 months. There has been no fever, malaise, or neck stiffness. Her mother died at a young age of “something neurological.” On examination, she has quick jerks in both arms and face. Which of the following is THE MOST LIKELY diagnosis?

A) Huntington disease

B) Creutzfeldt-Jacob disease C) Sydenham chorea

D) Infarct of the subthalamic nucleus E) Herpes encephalitis

27

—————————————————–Page 27—————————————————–

117 . Causes of hypoadrenalism include:

A) Hughes’ syndrome (anti-phospholipid antibody) B) MEN type 2a

C) VonHippel-Lindau

D) Penderd’s syndrome E) McArdle’s syndrome

118 . Which of the following is a recognized feature of massive pulmonary embolism?

A) Reduced plasma lactate levels

B) An increase in serum troponin levels C) An arterial pH less than 7.2

D) Blood gases show increased pCO2 on air E) Normal D-dimer levels

119. A 64-year-old man is found to have squamous cell bronchogenic carcinoma. Which of the following statements is true regarding surgical resection? A) An FEV1 of 2 L is a major contraindication to surgical resection. B) Hypercalcemia makes further assessment for surgery unnecessary.

C) Is precluded if a CT scan of the thorax shows enlarged mediastinal lymph nodes. D) Positive sputum cytology excludes the need for bronchoscopic examination of the

airways.

E) The presence of finger clubbing indicates that liver metastases are already present.

120. A 68-year-old woman complains of loss of appetite, weakness, fatigue, constipation, and impaired memory. She has a history of two episodes of nephrolithiasis. Laboratory evaluation reveals calcium levels and PTH are high. Which one of the following is a common manifestation of this disease?

A) Anxiety

B) Bone fractures C) Heart failure D) Hirsutism

E) Proximal muscle weakness

28

—————————————————–Page 28—————————————————–

B: For each question below determine which answer is true or false 121. Causes of acute pancreatitis include:

A) Alcohol (True) B) Hypocalcaemia C) Hyperlipidemia

D) Self-poisoning with diazepam

E) Endoscopic retrograde cholangiopancreatography (ERCP)

122. Fresh frozen plasma (FFP)

A) it contains all factors

B) intravenous administration can lead to volume overload C) viral transmission is possible with transfusion

D) it is a first-line treatment for uremic patients with bleeding

E) it is a first-line treatment for bleeding patients with a coagulopathy

123. A 35 year old man undergoes autologous bone marrow transplantation following

successful treatment of lymphoma. Two weeks later he develops scattered bilateral progressive breathlessness and dry cough. HRCT demonstrates several areas of bilateral ground glass changes with associated reticular changes, but no effusions. Which of the following is/are the most likely explanation?

A) Angioinvasive aspergillosis

B) Lymphoid interstitial pneumonia C) CMV pneumonia D) Drug toxicity

E) Pulmonary edema

124. Which of the following is/are associated with Hyperuricemia?

A) Is usually due to an excess purine consumption

B) Occurs in association with acute lymphoblastic leukemia C) In primary gout is inherited in an autosomal dominant manner D) Can be reduced with low dose aspirin therapy

E) Can be treated with uricosuric drugs even in renal failure

125. The following is/are complications of nephrotic syndrome

A) Acute renal failure

B) Accelerated hypertension C) Hypocalcaemia

D) Pneumococcal infection E) Venous thrombosis

126. A 45-year-old man develops weakness and wasting of the right hand. Which of the following is/are likely to be the cause?

A) Old injury to the elbow joint.

B) Bronchogenic carcinoma of the right upper lobe. C) Multiple sclerosis. D) Syringomyelia.

E) Motor neurone disease.

29

—————————————————–Page 29—————————————————–

127. A 40 year old has a routine chest radiograph as a part of pre-immigration work up. This demonstrates a mass on the left with loss of the upper left heart border. The descending aorta can, however, be seen despite the mass. The most likely location of the mass:

A) Apico-posterior segment B) Lingula

C) Anterior segment of the upper lobe

D) Posterior basal segment of the lower lobe E) Lateral basal segment of the lower lobe

128. In an investigation for lung malignancy, all of the following may produce a false positive result on a PET-CT

A) Pulmonary hamartoma B) Intra-lobar sequestration C) Tuberculosis D) Pneumonia E) Scarring

129. Reliable indicators of tissue oxygenation include the following:

A) Base deficit in arterial blood B) pH of arterial blood C) PO2 of arterial blood D) pH of venous blood E) PO2 of venous blood

130. Subarachnoid haemorrhage: A) Is precipitated by dehydration B) Can be detected on a CT scan C) Is more common in males

D) Causes an increase in intracranial pressure

E) Is associated with a maximal incidence of vasospasm from the third to the tenth day

after the bleed

131. Lateral popliteal nerve block (common peroneal nerve) can cause loss of:

A) Dorsiflexion of the great toe

B) Sensation between the hallux and the second toe C) Evasion of the foot

D) Plantar flexion of the foo E) Ankle jerk

132. ‘Blue Bloaters’ (patients with chronic obstructive pulmonary disease showing marked cyanosis and oedema) differ from ‘Pink Puffers’ (patients with chronic obstructive pulmonary disease showing dyspnoea but not cyanosis) by having a lower

A) Forced expiratory volume in one second. B) Peak expiratory flow rate. C) Arterial blood pH.

D) Sensitivity to carbon dioxide. E) Pulmonary arterial pressure.

30

—————————————————–Page 30—————————————————–

133. An asthmatic patient develops acute bronchospasm at the end of an operation for the repair of an umbilical hernia. The possible causes include: A) Use of neostigmine to reverse the neuromuscular blockade B) A light level of anesthesia

C) Irritation of the respiratory tree by the tracheal tube D) Use of isoflurane during the anesthetic

E) Inclusion of morphine in the preoperative medication

134. DC cardio-version is indicated for:

A) Supraventricular tachycardia B) Ventricular tachycardia

C) Premature atrial contraction D) Digitalis toxicity

E) Premature ventricular contraction

135. The following increase the risk of cauda equine syndrome (CES):

A) Old age

B) The use of epidural adrenaline C) The lithotomy position D) The use of barbotage E) The prone position

136. An elevated left hemidiaphragm can be caused by:

A) Left phrenic nerve paralysis B) Left lower lobe collapse C) Sinus inversus

D) Left pleural effusion E) Left pneumothorax

137. During anaesthesia a patient with Parkinson’s disease taking levodopa should not receive:

A) Enflurane B) Fentanyl C) Morphine D) Droperidol

E) Nitrous oxide

138. All of the following is/are a cause of clubbing finger :

A) Empyema

B) Mesothelioma

C) Bronchogenic carcinoma D) Cystic fibrosis E) COPD

139. In patients with the acquired immune deficiency syndrome (AIDS)

A) Neutrophils are more affected than lymphocytes.

B) Total white cell count is a better indicator of progression than any subset of white cells. C) Host DNA is incorporated into the human immunodeficiency (HIV) virus.

D) Occurrence in infancy results from transmission of infection rather than inheritance. E) There is increased risk of malignant tumour

31

—————————————————–Page 31—————————————————–

140. Crohn’s disease is more likely than ulcerative colitis if:

A) Crypt abscesses are seen in a bowel biopsy. B) The patient has pyoderma gangrenosa.

C) The patient has bowel hemorrhage sufficient to require transfusion. D) The sigmoidoscopy is normal. E) The patient has clubbing.

141. The following are recognized features of the sleep apnea syndrome:

A) An abnormal EEG.

B) Chronic hypercapnia. C) Arterial desaturation. D) Nocturnal enuresis.

E) Cyclical changes in the RR interval of 40-80 ns in a continuous ECG recording.

142. When patients seroconvert to HIV:

A) They are more infectious.

B) They may already have developed cytotoxic lymphocytes against the virally infected

cells.

C) Antibodies to HIV are unlikely to develop until 20-25 weeks after the onset of symptoms. D) Development of symptomatic primary infection suggests that a patient is more likely to

develop AIDS within 3-5 years.

E) The fall in CD4 lymphocytes is offset by a rise in CD8 positive cells, so patients do not

become lvmphopenic.

143. Which of the following statement is/are true concerning tissue typing and renal transplantation?

A) Siblings with the same biologic parents have at 25% chance of being human leukocyte

antigen (HLA) incompatible

B) By definition, a true biologic parent shares one HLA haplotype with each child C) In vitro cytotoxicity tests do not predict rejection in the transplant setting

D) MHC class II antigens, expressed on B lymphocytes and vascular endothelial cells, are

pivotal in the rejection process

E) Mandatory sharing of 6 antigen-matched cadaver kidneys with the best marched recipient will dramatically improve graft survival for nearly all transplant recipients

144. Which of the following statements is/are true concerning creatinine clearance? A) Creatinine clearance normally exceeds “true” measurements of glomerular filtration rate B) Cimetidine and trimethoprim can block the secretory component of creatinine clearance C) Urinary creatinine excretion is primarily influenced by the muscle mass in the Steady

state

D) The secretory component of creatinine excretion may become more apparent when

serum creatinine concentrations are elevated

E) Creatinine clearance is linearly related to the serum creatinine concentration

145. Which of the following is/are risk factors for bronchogenic carcinoma?

A) Pulmonary edema B) Pulmonary fibrosis C) Asbestos exposure D) Tobacco abuse

E) Chromium exposure

32

—————————————————–Page 32—————————————————–

146. Which statements about neurohumoral activation in heart failure are true?

A) Circulating norepinephrine levels correlate with survival in heart failure

B) Proinflammatory cytokines are elevated in patients with a symptomatic left ventricular

dysfunction but then diminish as severity of heart failure increases

C) Levels of natriuretic peptides are increased in heart failure, but response to this peptides

is diminished

D) Endothelin levels are increased in congestive heart failure and contribute to peripheral

and pulmonary vasoconstriction

E) Plasma renin levels are markedly diminished in severe congestive heart failure, resulting

in systemic hypotension

147. Which of the following about this cardiac imaging modalities are correct? A) Aortic dissection can be assessed by compute tomography, aortography, magnetic

resonance imaging, and echocardiography

B) Electron beam computed tomography (EBCT) visualizes coronary calcification and has

been used to screen as symptomatic individuals for coronary artery disease

C) Cardiac catheterization is mandatory for all patients with aortic valve disease who are

being considered for aortic valve replacement

D) Radionuclide angiography can reliably assess myocardial wall thickness, wall motion,

and global left ventricular ejection fraction

E) Magnetic resonance imaging can identify Cardiomyopathy resulting from

hemochromatosis

148. Which of the following statements is/are true about the epidemiology of pancreatic cancer?

A) Carcinoma of the pancreas is more, in men than women

B) Coffee consumption is a very strong risk factor in the development of pancreatic cancer C) The incidence of pancreatic cancer has increased recently

D) The vast majority of pancreatic cancers derived from the cells of the pancreatic duct E) With new surgical techniques, 5-year survival rate of pancreatic cancer now approaches

50%

149. A 28-year-old female student from Ecuador comes to the student health service

complaining of abdominal pain for the past 2-3 months. She describes the pain as being crampy and associated with increased flatus. She does not have nausea, vomiting, weight loss, or intolerance to any specific food. She has frequent loose bowel movements 4-6 per day and none at night. She admits she has stress and insomnia related to completing her graduate studies. On physical examination she has mild L LQ tenderness without rebound. A stool test for occult blood is negative. Which of the following statements is/are true?

A) Obtain colonoscopy

B) Obtain stool studies for ova and parasites C) Refer patients to gastroenterologist D) Refer patient for psychological testing E) Begin a therapeutic trial with psyllium

150. Which of the following statements regarding medical therapy of IBD is/are true? A) Topical medicine of mine erythema therapy is effective in treating distal procto-colitis B) Metronidazole is useful in inducing remission off active ulcerative colitis C) Elemental diet have therapeutic benefit in active Crohn’s disease D) Corticosteroids should be discontinued in pregnant woman with IBD E) Therapy with sulfasalazine may be limited by allergy to the sulfa moiety

33

—————————————————–Page 33—————————————————–

151. Phaeochromocytomas:

A) Are noradrenaline secreting tumours

B) Can occur anywhere along the sympathetic chain C) Are usually benign

D) Produce hypotension

E) Produce excessive amounts of adrenaline

152. Which of the following are considered to be extra intestinal manifestations of inflammatory bowel disease?

A) Ankylosing spondylitis

B) Sclera is scapular arising cholangitis uvulitis C) Uveitis

D) Erythema nodosum E) Large joint arthritis

153. Which of the following statements is/are true concerning the eradication of H pylori? A) Most individuals with nonulcer dyspepsia and H. pylori infection have symptomatic cures

when the organism is eradicated.

B) Elimination of H. pylori is associated with a very low rate of ulcer recurrence.

C) All patients with evidence of H. pylori infection should be treated to decrease the risk of

gastric cancer

D) Successful treatment can be documented by repeating serologic testing in 4-6 weeks E) Appropriate treatment with antibiotic therapy leads to H. pylori eradication in greater than

80% of cases

154. Concerning tuberculin skin testing:

A) Patients with tuberculous pericarditis are usually tuberculin positive B) Previous BCG vaccination usually results in a strongly positive reaction C) Pulmonary sarcoidosis gives a positive tuberculin test in about 30% of cases D) If the sputum microscopy and culture are negative for mycobacteria, pulmonary

tuberculosis can only be diagnosed if the tuberculin test is positive.

E) The test is usually negative in miliary tuberculosis

155. Subcutaneous nodules are a typical finding in:

A) Neurofibromatosis B) Hydatid disease C) Cysticercosis

D) Onchocerciasis E) Trichinosis

156. In enteric fever

A) Severe cases should receive high dose dexamethasone B) Ciprofloxacin is the treatment of choice for adults C) Jaundice is a recognised complication D) Myocarditis is a recognised complication

E) If perforation of the bowel occurs it should be managed conservatively because surgery

carries too high a mortality

34

—————————————————–Page 34—————————————————–

157. The following tumors match with the appropriate infective agent:

A) Cholangiocarcinoma— Clonorchis sinensis

B) Carcinoma of cervix (and vulva)— Human papilloma virus C) Squamous carcinoma of bladder— Schistosoma hematobium D) Burkitt’s lymphoma— Epstein-Barr virus

E) Squamous carcinoma of the penis— Human herpes virus 8

158. Hepatitis B .

A) Babies born to hepatitis B e antigen positive (Hbe Ag +ve) mothers should be given

active and passive immunization at birth

B) The complete vaccine series induces protective antibody levels in more than 95% of

infants, children and young adults.

C) Co-infection with delta virus may occur in intravenous drug abusers but occurs less

commonly in homosexuals

D) Super-infection with delta virus causes a clinical deterioration E) Is the major underlying cause of hepatocellular carcinoma

159. A woman who had not previously travelled abroad spent five days in Zambia. She went on safari on the fifth day and on the following day she became ill with fever and an erythematous rash. She is likely to be suffering from:

A) Hepatitis A B) Loa loa

C) Falciparum malaria D) Brucellosis

E) Hodgkin’s disease 160. Celiac disease

A) All patients who present with dermatitis herpetiformis have evidence of celiac disease on

intestinal mucosal biopsy, though it may not be clinically evident.

B) IgA antiendomysial antibodies are no longer recommended due to the lack of

standardization among laboratories.

C) IgA tissue transglutaminase (IgA tTG) antibody has 95% sensitivity and 95% specificity

for the diagnosis of celiac disease.

D) Antigliadin antibodies are highly recommended because of their higher sensitivity and

specificity.

E) An IgA level should be obtained in patients with a negative IgA tTG antibody when celiac disease is strongly suspected because up to 3% of patients with celiac disease have IgA deficiency.

161. Which of the following is/are potential side effects of erythropoietin therapy?

A) Red blood cell aplasia B) Hypotension

C) Iron deficiency anemia D) Seizures

E) Thrombosis

162. Regarding platelet abnormalities, which of the following is/are TRUE?

A) Bleeding complications may arise if platelets are < 50,000/µL

B) patients are at risk for spontaneous bleeding if platelet counts are < 10,000/µL C) When platelets drop below 10,000/µL, the patient should receive a platelet transfusion D) Patients with ITP respond well to platelet transfusion

E) Each unit of platelets transfused should raise the platelet count by about 10,000/µL

35

—————————————————–Page 35—————————————————–

163. Which of the following statements about lupus anticoagulant are CORRECT ?

A) It is an antiphospholipid antibody that interferes with coagulation

B) Most patients with lupus anticoagulant do not have lupus (SLE) or a clinical bleeding

disorder

C) Affected patients are at risk for arterial and venous thrombosis D) Affected women may have recurrent fetal loss E) it is associated with a normal PTT

164. The following are commonly found in reactive arthritis:

A) Enthesitis B) Alopecia

C) Oral ulceration D) Scleritis

E) Keratoderma blennorrhagia

165. The following statements are true:

A) Hypocalcaemia causes prolongation of the prothrombin time

B) The prothrombin time is a sensitive test of hepatocellular dysfunction

C) The activated partial thromboplastin time (APTT) is prolonged by unfractionated heparin

therapy

D) The effect of heparin is reversed by vitamin K

E) Deep venous thrombosis can be reliably diagnosed by measuring fibrin degradation

products (FDPs)

166. The following are true:

A) There is weakness of elbow extension in a crutch palsy

B) Wasting of the hypothenar eminence occurs in the carpal tunnel syndrome C) Abduction of the thumb is impaired in an ulnar nerve lesion

D) The index finger is hyperextended at the metacarpophalangeal (MCP) joint in an ulnar

nerve lesion

E) Sensation is lost over the whole of the back of the hand in radial nerve damage

167. The following are features of urinary infections in elderly people:

A) Patients usually complain of dysuria B) They may present with falls

C) They may present with constipation

D) Sterile pyuria is most likely caused by tuberculosis

E) Estrogen supplements may reduce their frequency in postmenopausal women

168. A 62-year-old man presents with 6 months history of progressive low back pain and fatigability. On examination there is tenderness to percussion over the lumbosacral spine. Laboratory data showed Hb 11.9 g/dl; WBC 5.200/mm3; Serum creatinine 1.3 mg/dl; Ser um ca 9.8 mg/dl; Serum ALP 130 U/L (N 30 -110). Pelvis x-ray revealed multiple osteolytic and sclerotic lesions of both hip bones. Which of the following is/are false?

A) Measure PSA

B) US guided transrectal biopsy of the prostate C) Isotope bone scans

D) Measurement of serum dihydrotestosterone E) Serum and urine protein electrophoresis

36

—————————————————–Page 36—————————————————–

169. A 24-year-old woman presents with two days history of fever, sore throat and marked malaise, she had hyperthyroid Grave’s disease diagnosed two months back and on therapy with carbimazole 40mg/day and propranolol 120mg per day. On P/E: P: 120/min, BP: 110/60, T: 39.2C. She had diffuse moderate enlargement of the thyroid gland. Labs: total leukocytes count of 900, Absolute granulocyte count 215, Hb 12.2, platelet count 234000/µl. Serum bilirubin is 1.1mg/dl and SGOT 12, SGPT 22. Appropriate management of this patient may include:

A) Administration if broad spectrum antibiotic B) Administration of high dose of prednisone C) Substitute propylthiouracil for carbimazole

D) Administration of granulocyte colony stimulating factor E) Administration of human immunoglobulin

170. Loss of ankle reflexes is associated with:

A) Multiple sclerosis B) Taboparesis C) Ataxia

D) Bulbar palsy

E) Subacute combined degeneration of the cord

171 . The addition of continuous positive airway pressure (CPAP) into the breathing system causes a reduction in the:

A) FRC

B) Static compliance of the lung C) Dynamic compliance of the lung D) Airway conductance E) Work of breathing

172. The immediate management of hyperthermia due to severe exercise includes:

A) Immediate cooling B) Aspirin

C) Sodium bicarbonate D) Chlorpromazine E) IV crystalloid

173. The following drugs counteract the effect of isoprenaline on the heart:

A) Propranolol B) Atropine

C) Diazoxide

D) Trimetaphan E) Nifedipine

174. Characteristic findings in acute respiratory failure associated with chronic bronchitis include:

A) A raised jugular venous pressure B) Muscular twitching C) Papilledema

D) A small volume pulse E) Cold extremities

37

—————————————————–Page 37—————————————————–

175. Which of the following features of an asthma attack are classified as ‘life threatening’ in the 2011 BTS asthma guideline? A) Inability to complete sentences in one breath. B) PaO2 of >8kPa. C) Silent chest.

D) PaCO2 >6kPa.

E) Peak expiratory flow rate (PEFR) <50% of predicted.

176. With regard to bleeding and coagulopathy in the critically ill patient:

A) If a platelet transfusion is indicated, 1 unit will raise the count by approximately 20 x

10 9 /L.

B) The principal constituents of cryoprecipitate include Factors VIII, XIII, vWF, fibronectin

and fibrinogen.

C) A suggested dose of fresh frozen plasma in the bleeding trauma patient with

coagulopathy is 30ml/kg.

D) Desmopressin at a dose of 0.3 µ g/kg is a useful treatment in patients with coagulopathy

related to uremia, cirrhosis and aspirin use.

E) At temperatures of 33-35°C, altered enzyme kinet ics equate to a 33% reduction in

normal clotting factors.

177. Regarding the hepatorenal syndrome (HRS):

A) It is commonly over-diagnosed in patients with cirrhotic liver disease. B) HRS Type 1 has the poorest outcome.

C) Kidneys from patients with HRS are suitable for transplantation. D) The condition is associated with splanchnic vasodilatation. E) Terlipressin must be given by infusion.

178. With regard to a patient with a neuromuscular disorder on the critical care unit: A) Potassium-sparing diuretics should be avoided in patients with hypokalaemic periodic

paralysis.

B) Suxamethonium use should be avoided in patients with myasthenia gravis. C) Patients with motor neurone disease typically require double the standard dose of

suxamethonium to provide optimum intubating conditions.

D) Local anesthesia can exacerbate symptoms of multiple sclerosis.

E) In Guillain-Barré syndrome, non-depolarizing neuromuscular blocking drugs may be

used, but should be significantly dose-reduced.

179. A 67-year-old male has a diagnosis of myasthenia gravis (MG). Which of the following medications should be avoided to reduce the risk of exacerbation?

A) Gentamicin. B) Paracetamol. C) Trimethoprim. D) Ciprofloxacin. E) Aspirin

180. Regarding the assessment of the pulse, which of the following is/are false?

A) The character should be assessed centrally. B) A slow rising pulse indicates mitral stenosis.

C) Pulsus alternans indicates poor left ventricular function. D) Rate, rhythm, volume and character should be assessed. E) The heart rate will increase during inspiration in sinus rhythm.

38

—————————————————–Page 38—————————————————–

C. Each slide followed by best of five question, Choose the best answer

181 . A 59-year-old woman presents to the Emergency Department complaining of severely painful skin lesions in the abdomen and thighs. Her medical history is significant for hypertension, hyperlipidemia, and end-stage renal disease. She has an arteriovenous fistula and undergoes hemodialysis 3 times weekly. She denies any recent medication changes or any illicit drug abuse, and denies ever taking warfarin. She is afebrile with a blood pressure of 162/96 mmHg and a heart rate of 96 beats per minute. On physical examination, there are areas of livedo reticularis over the abdomen with subcutaneous nodules and several necrotic ulcerations with eschar on both thighs ( Figure below ). Her distal pulses are 2+ in all four extremities.

A review of her laboratory records over the past few months shows an uptrending calcium- phosphate product . Which of the following is the most likely diagnosis?

A) Henoch–Schonlein purpura B) Calcific uremic arteriolopathy C) Necrotizing fasciitis

D) Deep venous thrombosis

E) Nephrogenic systemic fibrosis

39

—————————————————–Page 39—————————————————–

182 . A 32-year-old man is brought into the Emergency Department after he had a seizure. He has no history of a seizure disorder and there was no preceding trauma, new medications, or illicit drugs. On examination, his temperature is 38.9°C, blood pressure is 118/70 mmHg, heart rate is 90 beats per minute, and respiratory rate is 12 breaths per minute. He is lethargic and has a horizontal gaze palsy affecting the left eye. Appropriate laboratory values are sent, and a lumbar puncture and MRI ( Figure below ) are performed. The CSF studies show a large number of RBCs in each tube.

Which of the following therapies is most likely to be active against this condition?

A) Dexamethasone B) Vancomycin

C) Amphotericin B D) Acyclovir

E) Ceftriaxone

40

—————————————————–Page 40—————————————————–

183 . A 73-year-old nursing home resident complains of severe difficulty breathing. The nursing home staff reports that she has had several episodes of pneumonia since she has been there and is often delirious during the night. She has a history of hypertension, diabetes, dementia, and chronic cough with daily sputum production. Her medications include metformin,

hydrochlorothiazide, over-the-counter cough suppressants, and haloperidol as needed. She has a 10 pack/year smoking history and quit 30 years ago. She has a temperature of 38.3°C, blood pressure of 104/62 mmHg, heart rate of 91 beats per minute, respiratory rate of 26 breaths per minute, and oxygen saturation of 94% on room air. The patient’s CT scan is shown below ( Figure below ).

Which of the following is the most likely cause of this patient’s lung findings?

A) Smoking history

B) Infection with Pseudomonas C) Recurrent aspiration events

D) Antibodies against type IV collagen E) A&B

41

—————————————————–Page 41—————————————————–

184 . A 58-year-old woman is referred to an ophthalmologist because of vision loss. She describes previous episodes of acute vision impairment that gradually improved over days. An image of her retina is shown in ( Figure below )

If this patient were to undergo renal biopsy, what are the pathologic findings associated with this disease?

A) Nodular glomerulosclerosis, mesangial expansion, and basement membrane thickening B) Scattered crescentic lesions with fibrin and plasma proteins C) Vascular intimal thickening and hyaline deposition D) Subepithelial immune complex deposits

42

—————————————————–Page 42—————————————————–

185. A 78-year-old male smoker presents with a 6-month history of left neck pain and a 2-week history of slurred speech and difficulty swallowing.

Where is the lesion?

A) Right cranial nerve XI B) Left cranial nerve XI C) Right cranial nerve XII D) Left cranial nerve XII E) None of the above

43

—————————————————–Page 43—————————————————–

186 . This 54-year-old female has a pruritic area just medial to her left scapula. She is using back scratchers, rubbing against a door, applying moisturizer, topical steroids and nothing helps.

What is the most likely diagnosis?

A) Post herpetic neuralgia

B) Trigeminal trophic syndrome C) Notalgia paresthetica D) Brachioradial pruritus E) Meralgia paresthetica

44

—————————————————–Page 44—————————————————–

A 38-year-old man with a history of atopic dermatitis is mildly unwell. The painful eruption shown in Figure below has developed over the last 3 days.

187. What is this condition called?

A) Impetigo

B) Eczema herpeticum

C) Molluscum contagiosum D) Verruca vulgaris E) Contact dermatitis

188. What is it caused by? A) Herpes simplex infection B) Neoplastic proliferation C) Senile alloeosis D) Allergic reaction E) Idiopathic

189. What medication would you use for his condition?

A) Antibiotics

B) Corticosteroid C) Cauterization D) Aciclovir E) No action

45

—————————————————–Page 45—————————————————–

190 . A 45-year-old woman was seen in the Emergency department having developed weakness and stiffness of her legs. Her symptoms had been progressive over a number of months. She appeared to walk with a stiff gait and had had a number of falls. She had a history of joint pains affecting her hands and feet and had recently noticed pain in her back around the thoracic area. She had not noticed any change in her bowels, but had noticed increased urinary frequency. She had attended her GP eight months ago with painful visual disturbance in her right eye, which seemed to recover after three days. No diagnosis was made at the time. Otherwise, she was fit and well, was a non-smoker and did not drink alcohol. On examination she had multiple oral ulcers. Her blood pressure was 145/78 mmHg, pulse was 87 beats per minute and regular and she was apyrexial. Examination on the cranial nerves did not reveal any abnormalities and fundoscopy was entirely normal. Upper limb examination revealed deformities of both hands as shown. Tone, power, reflexes and sensation in the upper limbs were intact. Lower limb examination revealed marked increase in tone with pyramidal weakness of 3/5 in both legs and pathologically brisk reflexes with bilateral extensor plantar responses. There was also reduced sensation to pinprick bilaterally to the level of T4. A photograph of the patient’s hands is shown below.

Investigations revealed: Rheumatoid factor negative; Anti-nuclear antigen 1: 160; Extractable nuclear antigen negative. A lumbar puncture was performed and yielded the following data: Opening pressure 12 cm H2O (6 – 18); CSF protein 0.79 g/L (0.15 – 0.45); CSF white cell count 10 cells per mL (<5); CSF white cell differential 90% lymphocytes; CSF red cell cou nt 2 cells per mL (<5); CSF oligoclonal bands Positive. Serum oligoclonal bands Positive. A plain x ray of the hands did not reveal any bone deformity.

What is the likely diagnosis in this patient?

A) Multiple sclerosis

B) Myelopathy secondary to osteoarthritis

C) Myelopathy secondary to rheumatoid arthritis D) Reiter’s syndrome

E) Systemic lupus erythematosus

46

—————————————————–Page 46—————————————————–

191 . A 61-year-old patient with diabetes shows you this skin eruption which has developed over the course of four weeks on the dorsum of his foot.

What treatment would you offer him?

A) Biopsy

B) Dapsone

C) Observation

D) Surgical excision with a 2 cm excision margin E) Surgical excision with a 5 cm excision margin

47

—————————————————–Page 47—————————————————–

192 . A 31-year-old man develops an erythematous rash overnight:

Which one of the following conditions is most strongly associated with this type of rash?

A) Crohn’s disease B) Tuberculosis C) Sarcoidosis

D) Herpes simplex virus

E) Staphylococcal infections

48

—————————————————–Page 48—————————————————–

193 . A 65-year-old man is evaluated for progressive dyspnea on exertion that has occurred over the course of the past 3 months. His medical history is significant for an episode of necrotizing pancreatitis that resulted in multiorgan failure and acute respiratory distress syndrome. He required mechanical ventilation for 6 weeks prior to his recovery. He also has a history of 30 pack-years of tobacco, quitting 15 years previously. He is not known to have chronic obstructive pulmonary disease. On physical examination, a low-pitched inspiratory and expiratory wheeze is heard, loudes over the mid-chest area. On pulmonary function testing, the forced expiratory volume in 1 second (FEV 1 ) is 2.5 L (78% predicted), forced vital capacity (FVC) is 4.00 L (94% predicted), and FEV 1 /FVC ratio is 62.5%. The flow-volume curve is shown below. What is the most likely cause of the patient’s symptoms?

What is the most likely cause of the patient’s symptoms?

A) Aspirated foreign body

B) Chronic obstructive pulmonary disease C) Idiopathic pulmonary fibrosis D) Subglottic stenosis

E) Unilateral vocal cord paralysis

49

—————————————————–Page 49—————————————————–

194. You are asked to evaluate a 27-year-old internal medicine resident reporting 1 week of cough, coryza, and a low-grade fever. Today, he has developed rapidly escalating chest discomfort while in clinic. He notes that the pain becomes more intense when he takes a deep breath. You perform a standard 12-lead ECG (Figure below). On examination, his blood pressure is normal, he is afebrile, and his jugular venous pulse is not elevated. However, he appears mildly uncomfortable from the chest pain.

The next most appropriate step would be which of the following?

A) Administer aspirin, intravenous heparin, sublingual nitroglycerin, and clopidogrel. B) Emergently obtain transthoracic echocardiogram with possible pericardiocentesis. C) Emergently perform coronary angiography to evaluate for acute myocardial infarction. D) Prescribe ibuprofen and colchicine. E) Refer for treadmill stress test.

50

—————————————————–Page 50—————————————————–

A 74-year-old woman is 2 days status post hip surgery for a fracture after a fall. Her only medication prior to admission was a calcium supplement, and she has no prior surgical history. Over the past 24 hours, she has had increasing abdominal discomfort and distension. She received a dose of cefazolin prior to surgery but no other antibiotics. On physical examination, she is afebrile with blood pressure of 140/80 mmHg, heart rate of 110 bpm, respiratory rate of 16 breaths/min, and oxygen saturation of 100% on 2 L of nasal oxygen. She has a distended tympanic abdomen with absent bowel sounds. There is no rebound tenderness. Her upright abdominal film is shown below.

195. Which of the following is the most likely diagnosis?

A) Acalculous cholecystitis

B) Colonic pseudo-obstruction C) Perforated duodenal ulcer D) Small bowel obstruction E) Small bowel ileus

196. Which of the following is the next recommended therapy for this patient?

A) Atropine

B) Laparotomy C) Morphine

D) Neostigmine E) Vancomycin

51

—————————————————–Page 51—————————————————–

197. A 56-year-old male, presented to eye clinic for follow up. Fundus examination is shown below.

What is the primary finding seen in this patient’s fundus?

A) Arteriovenous nicking B) Microaneurysms

C) Neovascularization D) Papilledema

52

—————————————————–Page 52—————————————————–

198. A 39-year-old man comes to clinic complaining of blistering skin lesions on the backs of his hands and arms that are painful. They are often precipitated by sunlight and heal with scarring. He also notices that they often occur after drinking alcohol heavily. His hands and forearms have numerous hypopigmented scars that he says are from previous episodes. The skin over the back of his hands appears thick and coarse. Otherwise his review of systems and physical examination are normal. The lesions on his hands are shown below. Testing confirms your suspected diagnosis.

Which of the following treatments will most likely lead to long-term improvement for this patient?

A) Avoidance of sun exposure and IV hemin for treatment of acute lesions B) Hydroxychloroquine 200 mg twice daily

C) Phlebotomy of 450 mL of blood every 1–2 weeks D) Prednisone 0.5 mg/kg orally daily

E) Triamcinolone 0.5% topically twice daily

53

—————————————————–Page 53—————————————————–

199 . This 45-year-old male patient presented with abdominal pain. On physical exam at presentation to this hospital, the patient’s abdomen was soft and non-tender. Multiple erythematous ulcerated nodules were seen on his bilateral lower extremities extending to his thighs (Figure below). He had an elevated lipase level.

What is the diagnosis? A) Adenovirus infection B) Calciphylaxis

C) Erythema nodosum D) Panniculitis E) Tuberculosis

54

—————————————————–Page 54—————————————————–

200. A 68-year-old man with a 15-year history of redness of face whenever he tried to comb his hair in the mornings, visit a clinical for consultation. Patient also gave history suggestive of breathlessness, choking and stridor during these times. Patient was asked to raise his hands above his head and within 25 seconds of raising both the arms, he developed marked plethora of face, (Figure below). Physical examination revealed a diffusely enlarged thyroid with no palpable nodules, but the lower poles of the thyroid were not palpable. The patient’s serum thyrotropin, free thyroxine and calcitonin concentrations were normal.

Next appropriate step include

A) Non Contrast Computed tomography of the neck and chest B) Contrast-enhanced Computed tomography of the neck and chest C) MRI of the neck and chest D) Ultrasound of the neck

E) Repeat thyroid function test

Answer key in the next page, best luck

55

—————————————————–Page 55—————————————————–

Paper 8

I: Best answer II: false or true

1. B 2. C 3. D 4. E 5. B 6. C 7. E 8. E 9. B 10. D 11. D 12. E 13. E 14. E 15. C 16. B 17. C 18. A 19. C 20. D 21. C 22. D 23. C 24. B 25. C 26. E 27. C 28. D 29. A 30. A
31. D 32. D 33. C 34. D 35. C 36. A 37. D 38. E 39. A 40. D 41. A 42. B 43. C 44. A 45. C 46. E 47. E 48. A 49. D 50. D 51. A 52. D 53. C 54. B 55. B 56. A 57. A 58. C 59. B 60. A
61. C 62. B 63. D 64. B 65. B 66. E 67. A 68. C 69. D 70. E 71. B 72. C 73. D 74. A 75. B 76. B 77. D 78. A 79. B 80. D 81. D 82. C 83. E 84. B 85. B 86. C 87. B 88. D 89. C 90. B
91. E 92. A 93. D 94. E 95. B 96. E 97. C 98. C 99. C 100. C 101. A 102. E 103. D 104. D 105. B 106. A 107. E 108. D 109. E 110. B 111. A 112. E 113. C 114. C 115. B 116. A 117. A 118. B 119. C 120. B
121. TFTFT 122. TTTFT 123. FFFTF 124. FTFFF 125. TFTTT 126. TTFTT 127. FTFFF 128. TFTTT 129. TFFFT 130. FTFTT 131. TTTFF 132. FFTTF 133. TTTFF 134. TTFFF 135. TTFFF 136. TTTFT 137. FFFTF 138. TTTTF 139. FFFTT 140. FFFTT 141. FTTTT 142. TTFTF 143. TTFTF 144. TTTTF 145. FTTTT 146. TFTTT 147. TTFFT

148. TFTTF 149. FTFFT 150. TFTFT 151. TTTFT 152. TTTTT 153. FTTFT 154. TFTFT 155. TFTTF 156. TTTTF 157. TTTTF 158. TTTTT 159. FFFFF 160. TTTFT 161. TFTTT 162. TTTFT 163. TTTTF 164. TFTFT 165. FTTFF 166. TFFFF 167. FTTFT 168. TTTFT 169. TFFTF 170. FTFFT 171. FFFFT 172. TFFFT 173. TFFFT 174. TTTFF

175. FFTFF 176. TTFTT 177. TTTTF 178. FFFTT 179. TFFTF 180. TFTTT

III. Slides

181. B 182. D 183. C 184. A 185. D
186. C 187. B 188. A 189. D 190. E
191. C 192. D 193. D 194. D 195. B
196. D 197. C 198. C 199. D 200. A

56

—————————————————–Page 56—————————————————–

All books/videos/software featured here are free and NOT HOSTED ON OUR WEBSITE. If you feel that your copyrights have been violated, then please contact us immediately.